Why doesn't mathematics collapse down, even though humans quite often make mistakes in their proofs?Examples of common false beliefs in mathematicsHow do I fix someone's published error?Endless controversyWhen have we lost a body of mathematics because errors were found?How to be rigorous about combinatorial algorithms?How do you find out the latest news in fields other than your own?Advice on doing physics under the umbrella of mathematics and the converseDifferent styles of writing/reading articlesEndless controversyDid Hilbert laugh?What are examples of books which teach the practice of mathematics?Publishing a Simple Paper as an Undergraduate

Why doesn't mathematics collapse down, even though humans quite often make mistakes in their proofs?


Examples of common false beliefs in mathematicsHow do I fix someone's published error?Endless controversyWhen have we lost a body of mathematics because errors were found?How to be rigorous about combinatorial algorithms?How do you find out the latest news in fields other than your own?Advice on doing physics under the umbrella of mathematics and the converseDifferent styles of writing/reading articlesEndless controversyDid Hilbert laugh?What are examples of books which teach the practice of mathematics?Publishing a Simple Paper as an Undergraduate













48












$begingroup$


To begin with, I am aware of these questions, which seems to be related:
How do I fix someone's published error?, Examples of common false beliefs in mathematics, When have we lost a body of mathematics because errors were found?, etc...



My background: I am a senior undergraduate student in mathematics. Recently, I got a nice chance in a REU program, and started to read some journal articles. My impression was: any result in modern mathematics critically depends on another result, and that result depends on some other result, and ad infinitum.



On the other hand, some graduate students and professors in my university, who stand in quite intimate relations to me, say that, they do not check every details of proofs when they read mathematical monographs and research articles. They simply do not have enough time to read all the details and fill in the lines. (Clearly, I also do not read all the proofs in detail, if it seems to be so difficult or not much relevant to what I am interested in.)



Finally, I've been heard of some stories on fatal mathematical errors. To be honest, I do not understand what the errors precisely are. What I've been heard about are some "urban legends".
(I intentionally didn't write down the details of these urban legends, since if I write down everything I've heard, maybe someone working in the mentioned field may feel insulted...)



For the above reasons, recently I am afraid of the situation where a field in mathematics collapse down because of a single, fatal, but very subtle error in the foundations of that field. In mathematics, everything seems to be so much intertwined, and it seems that no one actually checks every single detail in every mathematical articles.



But the mathematics community seems to be very sound.
Maybe at least one of the followings are true:



  • Actually, a typical mathematical result does not depend that much on other results. So whenever if possible, a mathematician can check the details of every results which is of interest to him/her.


  • Strictly speaking, rigor is actually not that important. Even if a mathematical result turns out to be false, there is still something true in the statement. Therefore, only minor changes will be needed, and all the results depending on the turned-out-to-be-false result remains sound.


Here are my questions.



  1. Why the whole mathematics remains so sound, even though humans are imperfect and quite often produce errors? Are my explanations above correct?


  2. If a theorem turns out to be wrong, then mathematicians will try to correct (if possible) all the results depending on that theorem. How hard is this job? Isn't it very tedious and frustrating? I want to hear some personal stories.


  3. As an undergraduate student, I want to know if anybody who is much wiser, older, or experienced, had the same fear as mine. (Again, I want to hear some personal stories.)


  4. As an undergraduate student who will get into a graduate school in the near future, I want to get some advice. Should I stop worrying and believe the authors of the books and articles I read? When should I check all the details, and when should I just accept the theorem as given?


Thanks to everyone for reading my question.










share|cite|improve this question











$endgroup$









  • 2




    $begingroup$
    It's certainly possible that there are undiscovered errors which mess up whole series of results. The only cases we know for sure this hasn't happened are cases which are formally verified.
    $endgroup$
    – Will Sawin
    11 hours ago






  • 3




    $begingroup$
    If the alternative to collapse is that all published proofs are fully correct, then the collapse is already here. So what alternative to collapse do you want to explain: the persistence of funding for math, the reliability of math in engineering, the rarity of major errors in textbooks, ...?
    $endgroup$
    – Matt F.
    10 hours ago






  • 22




    $begingroup$
    It is not as if the dependence of one theorem on another goes one way; most mathematical bodies of knowledge consist of mutually re-inforcing theorems that collectively paint a picture about some class of mathematical object. Given the mutually reinforcing character of these theorems, a 'false' theorem is generally not likely to 'fit' with the remainder of mathematical knowledge properly, and will be spotted relatively quickly. Of course, that's just a general consideration, and there can certainly be exceptions. But it gives one way of thinking about the OP's question.
    $endgroup$
    – provocateur
    10 hours ago






  • 6




    $begingroup$
    @provocateur A problem with this argument is if we want our theorems to be proven/provable, and not just true, then the mutually reinforcing character is not so helpful - the theorem could reinforce the others because it is true, not because the proof is right.
    $endgroup$
    – Will Sawin
    8 hours ago






  • 9




    $begingroup$
    @WillSawin On the other hand, a "theorem" in the edifice of mathematical literature that is not fully proven, but is nevertheless true and mutually reinforcing with the rest of the edifice, is not particularly likely to cause a "collapse". Fixing such a "theorem" with an actual proof remains an important task, but not an existential one - the worst case is that the "theorem" gets demoted to a "widely believed conjecture".
    $endgroup$
    – Terry Tao
    7 hours ago
















48












$begingroup$


To begin with, I am aware of these questions, which seems to be related:
How do I fix someone's published error?, Examples of common false beliefs in mathematics, When have we lost a body of mathematics because errors were found?, etc...



My background: I am a senior undergraduate student in mathematics. Recently, I got a nice chance in a REU program, and started to read some journal articles. My impression was: any result in modern mathematics critically depends on another result, and that result depends on some other result, and ad infinitum.



On the other hand, some graduate students and professors in my university, who stand in quite intimate relations to me, say that, they do not check every details of proofs when they read mathematical monographs and research articles. They simply do not have enough time to read all the details and fill in the lines. (Clearly, I also do not read all the proofs in detail, if it seems to be so difficult or not much relevant to what I am interested in.)



Finally, I've been heard of some stories on fatal mathematical errors. To be honest, I do not understand what the errors precisely are. What I've been heard about are some "urban legends".
(I intentionally didn't write down the details of these urban legends, since if I write down everything I've heard, maybe someone working in the mentioned field may feel insulted...)



For the above reasons, recently I am afraid of the situation where a field in mathematics collapse down because of a single, fatal, but very subtle error in the foundations of that field. In mathematics, everything seems to be so much intertwined, and it seems that no one actually checks every single detail in every mathematical articles.



But the mathematics community seems to be very sound.
Maybe at least one of the followings are true:



  • Actually, a typical mathematical result does not depend that much on other results. So whenever if possible, a mathematician can check the details of every results which is of interest to him/her.


  • Strictly speaking, rigor is actually not that important. Even if a mathematical result turns out to be false, there is still something true in the statement. Therefore, only minor changes will be needed, and all the results depending on the turned-out-to-be-false result remains sound.


Here are my questions.



  1. Why the whole mathematics remains so sound, even though humans are imperfect and quite often produce errors? Are my explanations above correct?


  2. If a theorem turns out to be wrong, then mathematicians will try to correct (if possible) all the results depending on that theorem. How hard is this job? Isn't it very tedious and frustrating? I want to hear some personal stories.


  3. As an undergraduate student, I want to know if anybody who is much wiser, older, or experienced, had the same fear as mine. (Again, I want to hear some personal stories.)


  4. As an undergraduate student who will get into a graduate school in the near future, I want to get some advice. Should I stop worrying and believe the authors of the books and articles I read? When should I check all the details, and when should I just accept the theorem as given?


Thanks to everyone for reading my question.










share|cite|improve this question











$endgroup$









  • 2




    $begingroup$
    It's certainly possible that there are undiscovered errors which mess up whole series of results. The only cases we know for sure this hasn't happened are cases which are formally verified.
    $endgroup$
    – Will Sawin
    11 hours ago






  • 3




    $begingroup$
    If the alternative to collapse is that all published proofs are fully correct, then the collapse is already here. So what alternative to collapse do you want to explain: the persistence of funding for math, the reliability of math in engineering, the rarity of major errors in textbooks, ...?
    $endgroup$
    – Matt F.
    10 hours ago






  • 22




    $begingroup$
    It is not as if the dependence of one theorem on another goes one way; most mathematical bodies of knowledge consist of mutually re-inforcing theorems that collectively paint a picture about some class of mathematical object. Given the mutually reinforcing character of these theorems, a 'false' theorem is generally not likely to 'fit' with the remainder of mathematical knowledge properly, and will be spotted relatively quickly. Of course, that's just a general consideration, and there can certainly be exceptions. But it gives one way of thinking about the OP's question.
    $endgroup$
    – provocateur
    10 hours ago






  • 6




    $begingroup$
    @provocateur A problem with this argument is if we want our theorems to be proven/provable, and not just true, then the mutually reinforcing character is not so helpful - the theorem could reinforce the others because it is true, not because the proof is right.
    $endgroup$
    – Will Sawin
    8 hours ago






  • 9




    $begingroup$
    @WillSawin On the other hand, a "theorem" in the edifice of mathematical literature that is not fully proven, but is nevertheless true and mutually reinforcing with the rest of the edifice, is not particularly likely to cause a "collapse". Fixing such a "theorem" with an actual proof remains an important task, but not an existential one - the worst case is that the "theorem" gets demoted to a "widely believed conjecture".
    $endgroup$
    – Terry Tao
    7 hours ago














48












48








48


19



$begingroup$


To begin with, I am aware of these questions, which seems to be related:
How do I fix someone's published error?, Examples of common false beliefs in mathematics, When have we lost a body of mathematics because errors were found?, etc...



My background: I am a senior undergraduate student in mathematics. Recently, I got a nice chance in a REU program, and started to read some journal articles. My impression was: any result in modern mathematics critically depends on another result, and that result depends on some other result, and ad infinitum.



On the other hand, some graduate students and professors in my university, who stand in quite intimate relations to me, say that, they do not check every details of proofs when they read mathematical monographs and research articles. They simply do not have enough time to read all the details and fill in the lines. (Clearly, I also do not read all the proofs in detail, if it seems to be so difficult or not much relevant to what I am interested in.)



Finally, I've been heard of some stories on fatal mathematical errors. To be honest, I do not understand what the errors precisely are. What I've been heard about are some "urban legends".
(I intentionally didn't write down the details of these urban legends, since if I write down everything I've heard, maybe someone working in the mentioned field may feel insulted...)



For the above reasons, recently I am afraid of the situation where a field in mathematics collapse down because of a single, fatal, but very subtle error in the foundations of that field. In mathematics, everything seems to be so much intertwined, and it seems that no one actually checks every single detail in every mathematical articles.



But the mathematics community seems to be very sound.
Maybe at least one of the followings are true:



  • Actually, a typical mathematical result does not depend that much on other results. So whenever if possible, a mathematician can check the details of every results which is of interest to him/her.


  • Strictly speaking, rigor is actually not that important. Even if a mathematical result turns out to be false, there is still something true in the statement. Therefore, only minor changes will be needed, and all the results depending on the turned-out-to-be-false result remains sound.


Here are my questions.



  1. Why the whole mathematics remains so sound, even though humans are imperfect and quite often produce errors? Are my explanations above correct?


  2. If a theorem turns out to be wrong, then mathematicians will try to correct (if possible) all the results depending on that theorem. How hard is this job? Isn't it very tedious and frustrating? I want to hear some personal stories.


  3. As an undergraduate student, I want to know if anybody who is much wiser, older, or experienced, had the same fear as mine. (Again, I want to hear some personal stories.)


  4. As an undergraduate student who will get into a graduate school in the near future, I want to get some advice. Should I stop worrying and believe the authors of the books and articles I read? When should I check all the details, and when should I just accept the theorem as given?


Thanks to everyone for reading my question.










share|cite|improve this question











$endgroup$




To begin with, I am aware of these questions, which seems to be related:
How do I fix someone's published error?, Examples of common false beliefs in mathematics, When have we lost a body of mathematics because errors were found?, etc...



My background: I am a senior undergraduate student in mathematics. Recently, I got a nice chance in a REU program, and started to read some journal articles. My impression was: any result in modern mathematics critically depends on another result, and that result depends on some other result, and ad infinitum.



On the other hand, some graduate students and professors in my university, who stand in quite intimate relations to me, say that, they do not check every details of proofs when they read mathematical monographs and research articles. They simply do not have enough time to read all the details and fill in the lines. (Clearly, I also do not read all the proofs in detail, if it seems to be so difficult or not much relevant to what I am interested in.)



Finally, I've been heard of some stories on fatal mathematical errors. To be honest, I do not understand what the errors precisely are. What I've been heard about are some "urban legends".
(I intentionally didn't write down the details of these urban legends, since if I write down everything I've heard, maybe someone working in the mentioned field may feel insulted...)



For the above reasons, recently I am afraid of the situation where a field in mathematics collapse down because of a single, fatal, but very subtle error in the foundations of that field. In mathematics, everything seems to be so much intertwined, and it seems that no one actually checks every single detail in every mathematical articles.



But the mathematics community seems to be very sound.
Maybe at least one of the followings are true:



  • Actually, a typical mathematical result does not depend that much on other results. So whenever if possible, a mathematician can check the details of every results which is of interest to him/her.


  • Strictly speaking, rigor is actually not that important. Even if a mathematical result turns out to be false, there is still something true in the statement. Therefore, only minor changes will be needed, and all the results depending on the turned-out-to-be-false result remains sound.


Here are my questions.



  1. Why the whole mathematics remains so sound, even though humans are imperfect and quite often produce errors? Are my explanations above correct?


  2. If a theorem turns out to be wrong, then mathematicians will try to correct (if possible) all the results depending on that theorem. How hard is this job? Isn't it very tedious and frustrating? I want to hear some personal stories.


  3. As an undergraduate student, I want to know if anybody who is much wiser, older, or experienced, had the same fear as mine. (Again, I want to hear some personal stories.)


  4. As an undergraduate student who will get into a graduate school in the near future, I want to get some advice. Should I stop worrying and believe the authors of the books and articles I read? When should I check all the details, and when should I just accept the theorem as given?


Thanks to everyone for reading my question.







soft-question ho.history-overview advice






share|cite|improve this question















share|cite|improve this question













share|cite|improve this question




share|cite|improve this question








asked 11 hours ago


























community wiki





J. Doe











  • 2




    $begingroup$
    It's certainly possible that there are undiscovered errors which mess up whole series of results. The only cases we know for sure this hasn't happened are cases which are formally verified.
    $endgroup$
    – Will Sawin
    11 hours ago






  • 3




    $begingroup$
    If the alternative to collapse is that all published proofs are fully correct, then the collapse is already here. So what alternative to collapse do you want to explain: the persistence of funding for math, the reliability of math in engineering, the rarity of major errors in textbooks, ...?
    $endgroup$
    – Matt F.
    10 hours ago






  • 22




    $begingroup$
    It is not as if the dependence of one theorem on another goes one way; most mathematical bodies of knowledge consist of mutually re-inforcing theorems that collectively paint a picture about some class of mathematical object. Given the mutually reinforcing character of these theorems, a 'false' theorem is generally not likely to 'fit' with the remainder of mathematical knowledge properly, and will be spotted relatively quickly. Of course, that's just a general consideration, and there can certainly be exceptions. But it gives one way of thinking about the OP's question.
    $endgroup$
    – provocateur
    10 hours ago






  • 6




    $begingroup$
    @provocateur A problem with this argument is if we want our theorems to be proven/provable, and not just true, then the mutually reinforcing character is not so helpful - the theorem could reinforce the others because it is true, not because the proof is right.
    $endgroup$
    – Will Sawin
    8 hours ago






  • 9




    $begingroup$
    @WillSawin On the other hand, a "theorem" in the edifice of mathematical literature that is not fully proven, but is nevertheless true and mutually reinforcing with the rest of the edifice, is not particularly likely to cause a "collapse". Fixing such a "theorem" with an actual proof remains an important task, but not an existential one - the worst case is that the "theorem" gets demoted to a "widely believed conjecture".
    $endgroup$
    – Terry Tao
    7 hours ago













  • 2




    $begingroup$
    It's certainly possible that there are undiscovered errors which mess up whole series of results. The only cases we know for sure this hasn't happened are cases which are formally verified.
    $endgroup$
    – Will Sawin
    11 hours ago






  • 3




    $begingroup$
    If the alternative to collapse is that all published proofs are fully correct, then the collapse is already here. So what alternative to collapse do you want to explain: the persistence of funding for math, the reliability of math in engineering, the rarity of major errors in textbooks, ...?
    $endgroup$
    – Matt F.
    10 hours ago






  • 22




    $begingroup$
    It is not as if the dependence of one theorem on another goes one way; most mathematical bodies of knowledge consist of mutually re-inforcing theorems that collectively paint a picture about some class of mathematical object. Given the mutually reinforcing character of these theorems, a 'false' theorem is generally not likely to 'fit' with the remainder of mathematical knowledge properly, and will be spotted relatively quickly. Of course, that's just a general consideration, and there can certainly be exceptions. But it gives one way of thinking about the OP's question.
    $endgroup$
    – provocateur
    10 hours ago






  • 6




    $begingroup$
    @provocateur A problem with this argument is if we want our theorems to be proven/provable, and not just true, then the mutually reinforcing character is not so helpful - the theorem could reinforce the others because it is true, not because the proof is right.
    $endgroup$
    – Will Sawin
    8 hours ago






  • 9




    $begingroup$
    @WillSawin On the other hand, a "theorem" in the edifice of mathematical literature that is not fully proven, but is nevertheless true and mutually reinforcing with the rest of the edifice, is not particularly likely to cause a "collapse". Fixing such a "theorem" with an actual proof remains an important task, but not an existential one - the worst case is that the "theorem" gets demoted to a "widely believed conjecture".
    $endgroup$
    – Terry Tao
    7 hours ago








2




2




$begingroup$
It's certainly possible that there are undiscovered errors which mess up whole series of results. The only cases we know for sure this hasn't happened are cases which are formally verified.
$endgroup$
– Will Sawin
11 hours ago




$begingroup$
It's certainly possible that there are undiscovered errors which mess up whole series of results. The only cases we know for sure this hasn't happened are cases which are formally verified.
$endgroup$
– Will Sawin
11 hours ago




3




3




$begingroup$
If the alternative to collapse is that all published proofs are fully correct, then the collapse is already here. So what alternative to collapse do you want to explain: the persistence of funding for math, the reliability of math in engineering, the rarity of major errors in textbooks, ...?
$endgroup$
– Matt F.
10 hours ago




$begingroup$
If the alternative to collapse is that all published proofs are fully correct, then the collapse is already here. So what alternative to collapse do you want to explain: the persistence of funding for math, the reliability of math in engineering, the rarity of major errors in textbooks, ...?
$endgroup$
– Matt F.
10 hours ago




22




22




$begingroup$
It is not as if the dependence of one theorem on another goes one way; most mathematical bodies of knowledge consist of mutually re-inforcing theorems that collectively paint a picture about some class of mathematical object. Given the mutually reinforcing character of these theorems, a 'false' theorem is generally not likely to 'fit' with the remainder of mathematical knowledge properly, and will be spotted relatively quickly. Of course, that's just a general consideration, and there can certainly be exceptions. But it gives one way of thinking about the OP's question.
$endgroup$
– provocateur
10 hours ago




$begingroup$
It is not as if the dependence of one theorem on another goes one way; most mathematical bodies of knowledge consist of mutually re-inforcing theorems that collectively paint a picture about some class of mathematical object. Given the mutually reinforcing character of these theorems, a 'false' theorem is generally not likely to 'fit' with the remainder of mathematical knowledge properly, and will be spotted relatively quickly. Of course, that's just a general consideration, and there can certainly be exceptions. But it gives one way of thinking about the OP's question.
$endgroup$
– provocateur
10 hours ago




6




6




$begingroup$
@provocateur A problem with this argument is if we want our theorems to be proven/provable, and not just true, then the mutually reinforcing character is not so helpful - the theorem could reinforce the others because it is true, not because the proof is right.
$endgroup$
– Will Sawin
8 hours ago




$begingroup$
@provocateur A problem with this argument is if we want our theorems to be proven/provable, and not just true, then the mutually reinforcing character is not so helpful - the theorem could reinforce the others because it is true, not because the proof is right.
$endgroup$
– Will Sawin
8 hours ago




9




9




$begingroup$
@WillSawin On the other hand, a "theorem" in the edifice of mathematical literature that is not fully proven, but is nevertheless true and mutually reinforcing with the rest of the edifice, is not particularly likely to cause a "collapse". Fixing such a "theorem" with an actual proof remains an important task, but not an existential one - the worst case is that the "theorem" gets demoted to a "widely believed conjecture".
$endgroup$
– Terry Tao
7 hours ago





$begingroup$
@WillSawin On the other hand, a "theorem" in the edifice of mathematical literature that is not fully proven, but is nevertheless true and mutually reinforcing with the rest of the edifice, is not particularly likely to cause a "collapse". Fixing such a "theorem" with an actual proof remains an important task, but not an existential one - the worst case is that the "theorem" gets demoted to a "widely believed conjecture".
$endgroup$
– Terry Tao
7 hours ago











9 Answers
9






active

oldest

votes


















58












$begingroup$

In addition to the answers that have already been given, I think another reason that mathematics doesn't collapse is that the fundamental content of mathematics is ideas and understanding, not only proofs. If mathematics were done by computers that mindlessly searched for theorems and proof but sometimes made mistakes in their proofs, then I expect that it would collapse. But usually when a human mathematician proves a theorem, they do it by achieving some new understanding or idea, and usually that idea is "correct" even if the first proof given involving it is not.



One recent and well-publicized story is that told by the late Vladimir Voevodsky in his note The Origins and Motivations of Univalent Foundations. Here's a bit of one story that he tells about his own experience:




my paper "Cohomological Theory of Presheaves with Transfers," ... was written... in 1992-93. [Only] In 1999-2000... did I discover that the proof of a key lemma in my paper contained a mistake and that the lemma, as stated, could not be salvaged. Fortunately, I was able to prove a weaker and more complicated lemma, which turned out to be sufficient for all applications....



This story got me scared. Starting from 1993, multiple groups of mathematicians studied my paper at seminars and used it in their work and none of them noticed the mistake.... A technical argument by a trusted author, which is hard to check and looks similar to arguments known to be correct, is hardly ever checked in detail.




I don't know any of the details of the mathematics in this story, but the fact that he was able to prove a "weaker and more complicated lemma which turned out to be sufficient for all applications" matches my own experience. For instance, while working on a recent project I discovered no fewer than nine mistaken theorem statements (not just mistakes in proofs of correct theorems) in published or almost-published literature, including several by well-known experts (and two by myself). However, in all nine cases it was simple to strengthen the hypothesis or weaken the conclusion in such a way as to make the theorem true, in a way that sufficed for all the applications I know of.



I would argue that this is because the mistaken statements were based on correct ideas, and the mistakes were simply in making those ideas precise. Or to put it differently, mathematicians get our intuitions from "well-behaved" objects: sometimes that intuition can be wrong for "pathological" objects we didn't know about, but in such cases we simply alter the definitions to exclude the pathological ones from consideration.



On the other hand, people do sometimes get mistaken ideas. For instance, here's another quote from Voevodsky's article:




In October 1998, Carlos Simpson ... claimed to provide an argument that implied that the main result of the "∞-groupoids" paper, which Kapranov and I had published in 1989, cannot be true. However, Kapranov and I had considered a similar critique ourselves and had convinced each other that it did not apply. I was sure that we were right until the fall of 2013 (!!).



I can see two factors that contributed to this outrageous situation: Simpson claimed to have constructed a counterexample, but he was not able to show where the mistake was in our paper. Because of this, it was not clear whether we made a mistake somewhere in our paper or he made a mistake somewhere in his counterexample. Mathematical research currently relies on a complex system of mutual trust based on reputations. By the time Simpson’s paper appeared, both Kapranov and I had strong reputations. Simpson’s paper created doubts in our result, which led to it being unused by other researchers, but no one came forward and challenged us on it.




In this case I do know something about the mathematics involved, and my own opinion is somewhat different from Voevodsky's. In the 2000's I was a graduate student working on higher category theory, and my impression was that in the community of higher category theory it was taken for granted that Simpson's counterexample was correct and the Kapranov-Voevodsky paper was wrong, because the claimed KV result contradicted well-known ideas in the field.



The point here is that a community of people developing ideas together is likely to have arrived at correct intuitions, and these intuitions can flag "suspicious" results and lead to increased scrutiny of them. That is, when looking for mistaken ideas (as opposed to technical slips), it makes sense to give differing amounts of scrutiny to different claims based on whether they accord with the intuitions and expectations of experience.



So what do you do as a student? In addition to the other good advice that's been given, I think one of your primary goals should be to train your own intuition. That way you will be better-able to evaluate whether a given result, or something like it, is probably true, before you decide whether to read and check the proof in detail.



Of course, there is also the position that Voevodsky was led to:




And I now do my mathematics with a proof assistant. I have a lot of wishes in terms of getting this proof assistant to work better, but at least I don’t have to go home and worry about having made a mistake in my work.




I have a lot of respect for that position; I do plenty of formalization in proof assistants myself, and am very supportive of it. But I don't think that mathematics would be in danger of collapse without formalization, and I feel free to also do plenty of mathematics that would be prohibitively time-consuming to formalize in present-day proof assistants.






share|cite|improve this answer











$endgroup$














  • $begingroup$
    Great answer! Upvote.
    $endgroup$
    – Ivan Di Liberti
    8 hours ago






  • 8




    $begingroup$
    Agreed; very good answer. I particularly agree that that's probably how most mistakes get caught: not by nose-to-the-grindstone checks of logical correctness, but by having sensitive mathematical noses that detect when something smells just a little bit fishy. Human intuition is so much faster than human logic. Moreover, you have to develop the inner quietude to listen to those whisperings.
    $endgroup$
    – Todd Trimble
    8 hours ago






  • 3




    $begingroup$
    +1. I would also add that sometimes published, true mathematical results do contradict the community intuition and experiences; however, in these cases, mathematicians do check the published result very carefully, in order to check correctness and to update their intuition.
    $endgroup$
    – mathworker21
    5 hours ago











  • $begingroup$
    I have some discussion of these points at the end of my blog post terrytao.wordpress.com/career-advice/… . Mathematicians with a "post-rigorous" mindset can make formal errors, but these errors tend not to propagate all the way towards collapse, because of the soundness of the intuition guiding them.
    $endgroup$
    – Terry Tao
    3 hours ago










  • $begingroup$
    The classic and beautiful book Proofs and Refutations by Imre Lakatos discusses the nature of mathematical proofs (and their iterative refinement), using Euler's polyhedron formula $V-E+F=2$ as an example.You can be wrong, but still sort-of right.
    $endgroup$
    – Fab
    3 hours ago


















25












$begingroup$

  1. Redundancy is one big source of self-healing. A result with three different proofs is rather unlikely to be wrong. Also, people try to apply fresh results; wrong results often lead to contradictions when applied, alerting mathematicians to their wrongness. Same for proofs: Mistakes in proofs are often spotted when someone tries to adapt the proof to other questions.


  2. This is tricky. These days, using Google Scholar's "cited by" feature and various other backlink aggregators, you can get a list of papers/book that reference a given paper. Thus, if you find an error in the literature, you can track down where the "corruption" has spread. But getting corrections published is very difficult. Ted Hill and Nikolai Mnev are known for having struggled through the whole process of correcting someone else's false claims, but lots of people end up staying silent or (these days) just posting what they know somewhere on a forum like MathOverflow when someone stumbles upon the same problem. Then there are situations where no specific error can be pinpointed, but important material is simply imprecise and unreadable; fields often linger in such a limbo until someone does the thankless job of building the foundations underneath them. Katrin Wehrheim is one example of this.


  3. This question of mine got 41 votes, so yes, this is a fairly well-acknowledged problem.


  4. Ask your advisor and others. You definitely want to understand all proofs in undergraduate and lower-level graduate classes; they aren't particularly likely to be wrong, but you'll use the ideas anyway. As for advanced theory you rely upon, it depends.






share|cite|improve this answer











$endgroup$










  • 5




    $begingroup$
    For 4, I either agree and want to clarify or disagree slightly. You should understand all proofs presented in those classes, but you should trust the professor / the author about most of the theorems which are used in the classes but not proved, as sometimes happens when the proof of a result is too long and difficult or too far afield. It's definitely good to sometimes go beyond what is taught but in general the professor or textbook author probably decided what proofs to include based exactly on what will be useful to you in the future.
    $endgroup$
    – Will Sawin
    11 hours ago










  • $begingroup$
    Redundancy (1.) is good when the several proofs are by different authors. Several proofs in one paper, on the other hand, are not a good sign (as Arnol’d relished in pointing out).
    $endgroup$
    – Francois Ziegler
    51 mins ago



















16












$begingroup$

This is a broad question, but you may find it helpful to read The Existential Risk of Math Errors. It suggests a certain robustness of the mathematical edifice, which I actually think extends to the natural sciences as a whole. (Newtonian mechanics is "wrong" in a fundamental sense, but neither the development of relativistic mechanics nor the discovery of quantum mechanics has caused the collapse of classical mechanics.)



This quote in particular from Gian-Carlo Rota bears on your points 1 and 2:




When the Germans were planning to publish Hilbert’s collected papers
and to present him with a set on the occasion of one of his later
birthdays, they realized that they could not publish the papers in
their original versions because they were full of errors, some of them
quite serious. Thereupon they hired a young unemployed mathematician,
Olga Taussky-Todd, to go over Hilbert’s papers and correct all
mistakes. Olga labored for three years; it turned out that all
mistakes could be corrected without any major changes in the statement
of the theorems. There was one exception, a paper Hilbert wrote in his
old age, which could not be fixed; it was a purported proof of the
continuum hypothesis, you will find it in a volume of the
Mathematische Annalen of the early thirties. At last, on Hilbert’s
birthday, a freshly printed set of Hilbert’s collected papers was
presented to the Geheimrat. Hilbert leafed through them carefully and
did not notice anything.







share|cite|improve this answer











$endgroup$










  • 8




    $begingroup$
    Rota is unreliable here: Hilbert’s published theorems sometimes were incorrect as stated. See McLarty on the note in the Annalen about his famous result on invariants. webusers.imj-prg.fr/~michael.harris/theology.pdf
    $endgroup$
    – Matt F.
    10 hours ago







  • 11




    $begingroup$
    Rota is a great story-teller, but generally speaking people should fact-check him on his pronouncements.
    $endgroup$
    – Todd Trimble
    8 hours ago






  • 6




    $begingroup$
    @MattF: The word "major" in "major changes" is doing a lot of work, I suspect. An author's close collaborators usually have a good idea of what the author was meaning to say, and thus can fix all sorts of imprecisions that an outsider would be stumped by.
    $endgroup$
    – darij grinberg
    4 hours ago



















11












$begingroup$

If a result is not used much, then its veracity does not matter much for the rest of mathematics.



Otherwise, there may be several proofs of the result, which makes it much more probable (in a general sense) that the result is true. Importantly, there is usually some explanation or understanding of why the result is true, that is, the ideas behind the result.



Also, people try and do construct counterexamples to disprove a result, if they do not see why the result should be true.






share|cite|improve this answer











$endgroup$






















    11












    $begingroup$

    It might feel like the chain of citations goes ad infinitum,
    but of course this can't literally be the case. Indeed as long as
    mathematics is (at least primarily) an endeavor by and for humans,
    the entire chain of reasoning must be graspable by about the time a mathematician
    reaches PhD level. Much of it will not yet be accessible to an advanced
    college student such as the OP, and modern mathematics has gone in
    so many directions that no one mathematician can grasp more than
    a small sliver of the frontier $-$ which is why modern research mathematics
    requires specialization, and a mathematical community large enough to support
    such a large frontier. Inevitably errors occur, and a few of those
    propagate for some time before being caught. But the enterprise as a whole
    is self-correcting (as already explained in several ways in other answers).






    share|cite|improve this answer











    $endgroup$






















      5












      $begingroup$

      While the responses to this question have so far have generally (and correctly) focused on the robust nature of the edifice of mathematics as a whole, it might be worth pointing out by way of contrast that sometimes there are indeed doubts about foundational issues within a specific subfield (because, e.g., an "important" paper is known to have flaws), and that can be extremely deleterious to the field in question. Something along these lines is discussed in this Quanta magazine article: https://www.quantamagazine.org/the-fight-to-fix-symplectic-geometry-20170209/. (And I would say that what is described there is not even the most extreme example of what can happen.)






      share|cite|improve this answer











      $endgroup$










      • 1




        $begingroup$
        (This case is also mentioned by @darijgrinberg.)
        $endgroup$
        – Francois Ziegler
        1 hour ago



















      2












      $begingroup$

      Further to M. Shulman’s advice to “develop your intuition”, it’s probably worth adding that this is often done by understanding many examples, special cases whose moving parts are already transparent to you. You get something simpler and more robust. And psychologically at least, “trust” in a result often relies on familiarity with a library of such cases, more than on line-checking a general proof. (I would also guess that many innocuous “errors” only reflect some overshoot in generality when streamlining things for publication, and that is why not everything collapses.) Or, in the polemical words of Arnol’d (2004):




      There are two principal ways to formulate mathematical assertions (problems, conjectures, theorems,...): Russian and French. The Russian way is to choose the most simple and specific case (so that nobody could simplify the formulation preserving the main point). The French way is to generalize the statement as far as nobody could generalize it further.




      (That’s not to say that “top-down” never wins — e.g. reputedly, only the discovery of Bott periodicity settled a “spirited controversy” on specific homotopy groups (1959, p. 355 and Math review).)






      share|cite|improve this answer











      $endgroup$






















        0












        $begingroup$

        Well, because of a multitude of reasons, including:



        1. Mathematicians rarely make mistakes in simple proofs. And a lot of the basics of math are simple theorems that follow easily from the formal definitions.


        2. Mistakes get corrected quickly before claims get accepted and used:



          2.1 Most people double-check and re-check their own proofs before claiming to have proved something significant (although some don't - I've heard a rumor that Saharon Shelah had a bunch of papers with errors and brushed this off by saying they could be be easily rectified)



          2.2 Mathematicians are a community and check each other's work - so there's much less chance of mistakes escaping scrutiny.



        3. Doubt: People - mathematicians and users of math - don't just accept new claims at face value. Even if they don't have the time to check proofs themselves, they'll treat new claims/results as somewhat suspect until sufficient (or apparently sufficient) verification happens.


        4. Using an erroneous theorem usually leads to obvious problems, so whoever adopts an erroneous theorem as valid typically falls flat on their face, and this easily leads to doubt being cast on their assumptions - namely, the faulty theorem.



        5. New mistakes don't invalidate past work. If a field of mathematics has some valid, no-mistakes basis, then our mistakes building on that basis don't invalidate it. At most, we may get confused and doubt some of it - but that will only make us double-check its validity or look for a counter-example - which will fail.





        share|cite|improve this answer











        $endgroup$






















          -1












          $begingroup$

          Sometimes these collapses happen. Ole Peters and Murray Gell-Mann have a 2016 paper about a mistake in Expected Utility Theory, which they track down to a mistake in a Bernoulli paper from 1738.



          I'm an outsider looking in, but it sure looks like much of that entire field is in serious trouble. There's a post on this here: https://ergodicityeconomics.com/2018/02/16/the-trouble-with-bernoulli-1738/






          share|cite|improve this answer











          $endgroup$














          • $begingroup$
            While I know almost nothing about economics, my impression is that this critique is not accepted by mainstream economists.
            $endgroup$
            – Andy Putman
            2 hours ago













          Your Answer








          StackExchange.ready(function()
          var channelOptions =
          tags: "".split(" "),
          id: "504"
          ;
          initTagRenderer("".split(" "), "".split(" "), channelOptions);

          StackExchange.using("externalEditor", function()
          // Have to fire editor after snippets, if snippets enabled
          if (StackExchange.settings.snippets.snippetsEnabled)
          StackExchange.using("snippets", function()
          createEditor();
          );

          else
          createEditor();

          );

          function createEditor()
          StackExchange.prepareEditor(
          heartbeatType: 'answer',
          autoActivateHeartbeat: false,
          convertImagesToLinks: true,
          noModals: true,
          showLowRepImageUploadWarning: true,
          reputationToPostImages: 10,
          bindNavPrevention: true,
          postfix: "",
          imageUploader:
          brandingHtml: "Powered by u003ca class="icon-imgur-white" href="https://imgur.com/"u003eu003c/au003e",
          contentPolicyHtml: "User contributions licensed under u003ca href="https://creativecommons.org/licenses/by-sa/3.0/"u003ecc by-sa 3.0 with attribution requiredu003c/au003e u003ca href="https://stackoverflow.com/legal/content-policy"u003e(content policy)u003c/au003e",
          allowUrls: true
          ,
          noCode: true, onDemand: true,
          discardSelector: ".discard-answer"
          ,immediatelyShowMarkdownHelp:true
          );



          );













          draft saved

          draft discarded


















          StackExchange.ready(
          function ()
          StackExchange.openid.initPostLogin('.new-post-login', 'https%3a%2f%2fmathoverflow.net%2fquestions%2f338607%2fwhy-doesnt-mathematics-collapse-down-even-though-humans-quite-often-make-mista%23new-answer', 'question_page');

          );

          Post as a guest















          Required, but never shown

























          9 Answers
          9






          active

          oldest

          votes








          9 Answers
          9






          active

          oldest

          votes









          active

          oldest

          votes






          active

          oldest

          votes









          58












          $begingroup$

          In addition to the answers that have already been given, I think another reason that mathematics doesn't collapse is that the fundamental content of mathematics is ideas and understanding, not only proofs. If mathematics were done by computers that mindlessly searched for theorems and proof but sometimes made mistakes in their proofs, then I expect that it would collapse. But usually when a human mathematician proves a theorem, they do it by achieving some new understanding or idea, and usually that idea is "correct" even if the first proof given involving it is not.



          One recent and well-publicized story is that told by the late Vladimir Voevodsky in his note The Origins and Motivations of Univalent Foundations. Here's a bit of one story that he tells about his own experience:




          my paper "Cohomological Theory of Presheaves with Transfers," ... was written... in 1992-93. [Only] In 1999-2000... did I discover that the proof of a key lemma in my paper contained a mistake and that the lemma, as stated, could not be salvaged. Fortunately, I was able to prove a weaker and more complicated lemma, which turned out to be sufficient for all applications....



          This story got me scared. Starting from 1993, multiple groups of mathematicians studied my paper at seminars and used it in their work and none of them noticed the mistake.... A technical argument by a trusted author, which is hard to check and looks similar to arguments known to be correct, is hardly ever checked in detail.




          I don't know any of the details of the mathematics in this story, but the fact that he was able to prove a "weaker and more complicated lemma which turned out to be sufficient for all applications" matches my own experience. For instance, while working on a recent project I discovered no fewer than nine mistaken theorem statements (not just mistakes in proofs of correct theorems) in published or almost-published literature, including several by well-known experts (and two by myself). However, in all nine cases it was simple to strengthen the hypothesis or weaken the conclusion in such a way as to make the theorem true, in a way that sufficed for all the applications I know of.



          I would argue that this is because the mistaken statements were based on correct ideas, and the mistakes were simply in making those ideas precise. Or to put it differently, mathematicians get our intuitions from "well-behaved" objects: sometimes that intuition can be wrong for "pathological" objects we didn't know about, but in such cases we simply alter the definitions to exclude the pathological ones from consideration.



          On the other hand, people do sometimes get mistaken ideas. For instance, here's another quote from Voevodsky's article:




          In October 1998, Carlos Simpson ... claimed to provide an argument that implied that the main result of the "∞-groupoids" paper, which Kapranov and I had published in 1989, cannot be true. However, Kapranov and I had considered a similar critique ourselves and had convinced each other that it did not apply. I was sure that we were right until the fall of 2013 (!!).



          I can see two factors that contributed to this outrageous situation: Simpson claimed to have constructed a counterexample, but he was not able to show where the mistake was in our paper. Because of this, it was not clear whether we made a mistake somewhere in our paper or he made a mistake somewhere in his counterexample. Mathematical research currently relies on a complex system of mutual trust based on reputations. By the time Simpson’s paper appeared, both Kapranov and I had strong reputations. Simpson’s paper created doubts in our result, which led to it being unused by other researchers, but no one came forward and challenged us on it.




          In this case I do know something about the mathematics involved, and my own opinion is somewhat different from Voevodsky's. In the 2000's I was a graduate student working on higher category theory, and my impression was that in the community of higher category theory it was taken for granted that Simpson's counterexample was correct and the Kapranov-Voevodsky paper was wrong, because the claimed KV result contradicted well-known ideas in the field.



          The point here is that a community of people developing ideas together is likely to have arrived at correct intuitions, and these intuitions can flag "suspicious" results and lead to increased scrutiny of them. That is, when looking for mistaken ideas (as opposed to technical slips), it makes sense to give differing amounts of scrutiny to different claims based on whether they accord with the intuitions and expectations of experience.



          So what do you do as a student? In addition to the other good advice that's been given, I think one of your primary goals should be to train your own intuition. That way you will be better-able to evaluate whether a given result, or something like it, is probably true, before you decide whether to read and check the proof in detail.



          Of course, there is also the position that Voevodsky was led to:




          And I now do my mathematics with a proof assistant. I have a lot of wishes in terms of getting this proof assistant to work better, but at least I don’t have to go home and worry about having made a mistake in my work.




          I have a lot of respect for that position; I do plenty of formalization in proof assistants myself, and am very supportive of it. But I don't think that mathematics would be in danger of collapse without formalization, and I feel free to also do plenty of mathematics that would be prohibitively time-consuming to formalize in present-day proof assistants.






          share|cite|improve this answer











          $endgroup$














          • $begingroup$
            Great answer! Upvote.
            $endgroup$
            – Ivan Di Liberti
            8 hours ago






          • 8




            $begingroup$
            Agreed; very good answer. I particularly agree that that's probably how most mistakes get caught: not by nose-to-the-grindstone checks of logical correctness, but by having sensitive mathematical noses that detect when something smells just a little bit fishy. Human intuition is so much faster than human logic. Moreover, you have to develop the inner quietude to listen to those whisperings.
            $endgroup$
            – Todd Trimble
            8 hours ago






          • 3




            $begingroup$
            +1. I would also add that sometimes published, true mathematical results do contradict the community intuition and experiences; however, in these cases, mathematicians do check the published result very carefully, in order to check correctness and to update their intuition.
            $endgroup$
            – mathworker21
            5 hours ago











          • $begingroup$
            I have some discussion of these points at the end of my blog post terrytao.wordpress.com/career-advice/… . Mathematicians with a "post-rigorous" mindset can make formal errors, but these errors tend not to propagate all the way towards collapse, because of the soundness of the intuition guiding them.
            $endgroup$
            – Terry Tao
            3 hours ago










          • $begingroup$
            The classic and beautiful book Proofs and Refutations by Imre Lakatos discusses the nature of mathematical proofs (and their iterative refinement), using Euler's polyhedron formula $V-E+F=2$ as an example.You can be wrong, but still sort-of right.
            $endgroup$
            – Fab
            3 hours ago















          58












          $begingroup$

          In addition to the answers that have already been given, I think another reason that mathematics doesn't collapse is that the fundamental content of mathematics is ideas and understanding, not only proofs. If mathematics were done by computers that mindlessly searched for theorems and proof but sometimes made mistakes in their proofs, then I expect that it would collapse. But usually when a human mathematician proves a theorem, they do it by achieving some new understanding or idea, and usually that idea is "correct" even if the first proof given involving it is not.



          One recent and well-publicized story is that told by the late Vladimir Voevodsky in his note The Origins and Motivations of Univalent Foundations. Here's a bit of one story that he tells about his own experience:




          my paper "Cohomological Theory of Presheaves with Transfers," ... was written... in 1992-93. [Only] In 1999-2000... did I discover that the proof of a key lemma in my paper contained a mistake and that the lemma, as stated, could not be salvaged. Fortunately, I was able to prove a weaker and more complicated lemma, which turned out to be sufficient for all applications....



          This story got me scared. Starting from 1993, multiple groups of mathematicians studied my paper at seminars and used it in their work and none of them noticed the mistake.... A technical argument by a trusted author, which is hard to check and looks similar to arguments known to be correct, is hardly ever checked in detail.




          I don't know any of the details of the mathematics in this story, but the fact that he was able to prove a "weaker and more complicated lemma which turned out to be sufficient for all applications" matches my own experience. For instance, while working on a recent project I discovered no fewer than nine mistaken theorem statements (not just mistakes in proofs of correct theorems) in published or almost-published literature, including several by well-known experts (and two by myself). However, in all nine cases it was simple to strengthen the hypothesis or weaken the conclusion in such a way as to make the theorem true, in a way that sufficed for all the applications I know of.



          I would argue that this is because the mistaken statements were based on correct ideas, and the mistakes were simply in making those ideas precise. Or to put it differently, mathematicians get our intuitions from "well-behaved" objects: sometimes that intuition can be wrong for "pathological" objects we didn't know about, but in such cases we simply alter the definitions to exclude the pathological ones from consideration.



          On the other hand, people do sometimes get mistaken ideas. For instance, here's another quote from Voevodsky's article:




          In October 1998, Carlos Simpson ... claimed to provide an argument that implied that the main result of the "∞-groupoids" paper, which Kapranov and I had published in 1989, cannot be true. However, Kapranov and I had considered a similar critique ourselves and had convinced each other that it did not apply. I was sure that we were right until the fall of 2013 (!!).



          I can see two factors that contributed to this outrageous situation: Simpson claimed to have constructed a counterexample, but he was not able to show where the mistake was in our paper. Because of this, it was not clear whether we made a mistake somewhere in our paper or he made a mistake somewhere in his counterexample. Mathematical research currently relies on a complex system of mutual trust based on reputations. By the time Simpson’s paper appeared, both Kapranov and I had strong reputations. Simpson’s paper created doubts in our result, which led to it being unused by other researchers, but no one came forward and challenged us on it.




          In this case I do know something about the mathematics involved, and my own opinion is somewhat different from Voevodsky's. In the 2000's I was a graduate student working on higher category theory, and my impression was that in the community of higher category theory it was taken for granted that Simpson's counterexample was correct and the Kapranov-Voevodsky paper was wrong, because the claimed KV result contradicted well-known ideas in the field.



          The point here is that a community of people developing ideas together is likely to have arrived at correct intuitions, and these intuitions can flag "suspicious" results and lead to increased scrutiny of them. That is, when looking for mistaken ideas (as opposed to technical slips), it makes sense to give differing amounts of scrutiny to different claims based on whether they accord with the intuitions and expectations of experience.



          So what do you do as a student? In addition to the other good advice that's been given, I think one of your primary goals should be to train your own intuition. That way you will be better-able to evaluate whether a given result, or something like it, is probably true, before you decide whether to read and check the proof in detail.



          Of course, there is also the position that Voevodsky was led to:




          And I now do my mathematics with a proof assistant. I have a lot of wishes in terms of getting this proof assistant to work better, but at least I don’t have to go home and worry about having made a mistake in my work.




          I have a lot of respect for that position; I do plenty of formalization in proof assistants myself, and am very supportive of it. But I don't think that mathematics would be in danger of collapse without formalization, and I feel free to also do plenty of mathematics that would be prohibitively time-consuming to formalize in present-day proof assistants.






          share|cite|improve this answer











          $endgroup$














          • $begingroup$
            Great answer! Upvote.
            $endgroup$
            – Ivan Di Liberti
            8 hours ago






          • 8




            $begingroup$
            Agreed; very good answer. I particularly agree that that's probably how most mistakes get caught: not by nose-to-the-grindstone checks of logical correctness, but by having sensitive mathematical noses that detect when something smells just a little bit fishy. Human intuition is so much faster than human logic. Moreover, you have to develop the inner quietude to listen to those whisperings.
            $endgroup$
            – Todd Trimble
            8 hours ago






          • 3




            $begingroup$
            +1. I would also add that sometimes published, true mathematical results do contradict the community intuition and experiences; however, in these cases, mathematicians do check the published result very carefully, in order to check correctness and to update their intuition.
            $endgroup$
            – mathworker21
            5 hours ago











          • $begingroup$
            I have some discussion of these points at the end of my blog post terrytao.wordpress.com/career-advice/… . Mathematicians with a "post-rigorous" mindset can make formal errors, but these errors tend not to propagate all the way towards collapse, because of the soundness of the intuition guiding them.
            $endgroup$
            – Terry Tao
            3 hours ago










          • $begingroup$
            The classic and beautiful book Proofs and Refutations by Imre Lakatos discusses the nature of mathematical proofs (and their iterative refinement), using Euler's polyhedron formula $V-E+F=2$ as an example.You can be wrong, but still sort-of right.
            $endgroup$
            – Fab
            3 hours ago













          58












          58








          58





          $begingroup$

          In addition to the answers that have already been given, I think another reason that mathematics doesn't collapse is that the fundamental content of mathematics is ideas and understanding, not only proofs. If mathematics were done by computers that mindlessly searched for theorems and proof but sometimes made mistakes in their proofs, then I expect that it would collapse. But usually when a human mathematician proves a theorem, they do it by achieving some new understanding or idea, and usually that idea is "correct" even if the first proof given involving it is not.



          One recent and well-publicized story is that told by the late Vladimir Voevodsky in his note The Origins and Motivations of Univalent Foundations. Here's a bit of one story that he tells about his own experience:




          my paper "Cohomological Theory of Presheaves with Transfers," ... was written... in 1992-93. [Only] In 1999-2000... did I discover that the proof of a key lemma in my paper contained a mistake and that the lemma, as stated, could not be salvaged. Fortunately, I was able to prove a weaker and more complicated lemma, which turned out to be sufficient for all applications....



          This story got me scared. Starting from 1993, multiple groups of mathematicians studied my paper at seminars and used it in their work and none of them noticed the mistake.... A technical argument by a trusted author, which is hard to check and looks similar to arguments known to be correct, is hardly ever checked in detail.




          I don't know any of the details of the mathematics in this story, but the fact that he was able to prove a "weaker and more complicated lemma which turned out to be sufficient for all applications" matches my own experience. For instance, while working on a recent project I discovered no fewer than nine mistaken theorem statements (not just mistakes in proofs of correct theorems) in published or almost-published literature, including several by well-known experts (and two by myself). However, in all nine cases it was simple to strengthen the hypothesis or weaken the conclusion in such a way as to make the theorem true, in a way that sufficed for all the applications I know of.



          I would argue that this is because the mistaken statements were based on correct ideas, and the mistakes were simply in making those ideas precise. Or to put it differently, mathematicians get our intuitions from "well-behaved" objects: sometimes that intuition can be wrong for "pathological" objects we didn't know about, but in such cases we simply alter the definitions to exclude the pathological ones from consideration.



          On the other hand, people do sometimes get mistaken ideas. For instance, here's another quote from Voevodsky's article:




          In October 1998, Carlos Simpson ... claimed to provide an argument that implied that the main result of the "∞-groupoids" paper, which Kapranov and I had published in 1989, cannot be true. However, Kapranov and I had considered a similar critique ourselves and had convinced each other that it did not apply. I was sure that we were right until the fall of 2013 (!!).



          I can see two factors that contributed to this outrageous situation: Simpson claimed to have constructed a counterexample, but he was not able to show where the mistake was in our paper. Because of this, it was not clear whether we made a mistake somewhere in our paper or he made a mistake somewhere in his counterexample. Mathematical research currently relies on a complex system of mutual trust based on reputations. By the time Simpson’s paper appeared, both Kapranov and I had strong reputations. Simpson’s paper created doubts in our result, which led to it being unused by other researchers, but no one came forward and challenged us on it.




          In this case I do know something about the mathematics involved, and my own opinion is somewhat different from Voevodsky's. In the 2000's I was a graduate student working on higher category theory, and my impression was that in the community of higher category theory it was taken for granted that Simpson's counterexample was correct and the Kapranov-Voevodsky paper was wrong, because the claimed KV result contradicted well-known ideas in the field.



          The point here is that a community of people developing ideas together is likely to have arrived at correct intuitions, and these intuitions can flag "suspicious" results and lead to increased scrutiny of them. That is, when looking for mistaken ideas (as opposed to technical slips), it makes sense to give differing amounts of scrutiny to different claims based on whether they accord with the intuitions and expectations of experience.



          So what do you do as a student? In addition to the other good advice that's been given, I think one of your primary goals should be to train your own intuition. That way you will be better-able to evaluate whether a given result, or something like it, is probably true, before you decide whether to read and check the proof in detail.



          Of course, there is also the position that Voevodsky was led to:




          And I now do my mathematics with a proof assistant. I have a lot of wishes in terms of getting this proof assistant to work better, but at least I don’t have to go home and worry about having made a mistake in my work.




          I have a lot of respect for that position; I do plenty of formalization in proof assistants myself, and am very supportive of it. But I don't think that mathematics would be in danger of collapse without formalization, and I feel free to also do plenty of mathematics that would be prohibitively time-consuming to formalize in present-day proof assistants.






          share|cite|improve this answer











          $endgroup$



          In addition to the answers that have already been given, I think another reason that mathematics doesn't collapse is that the fundamental content of mathematics is ideas and understanding, not only proofs. If mathematics were done by computers that mindlessly searched for theorems and proof but sometimes made mistakes in their proofs, then I expect that it would collapse. But usually when a human mathematician proves a theorem, they do it by achieving some new understanding or idea, and usually that idea is "correct" even if the first proof given involving it is not.



          One recent and well-publicized story is that told by the late Vladimir Voevodsky in his note The Origins and Motivations of Univalent Foundations. Here's a bit of one story that he tells about his own experience:




          my paper "Cohomological Theory of Presheaves with Transfers," ... was written... in 1992-93. [Only] In 1999-2000... did I discover that the proof of a key lemma in my paper contained a mistake and that the lemma, as stated, could not be salvaged. Fortunately, I was able to prove a weaker and more complicated lemma, which turned out to be sufficient for all applications....



          This story got me scared. Starting from 1993, multiple groups of mathematicians studied my paper at seminars and used it in their work and none of them noticed the mistake.... A technical argument by a trusted author, which is hard to check and looks similar to arguments known to be correct, is hardly ever checked in detail.




          I don't know any of the details of the mathematics in this story, but the fact that he was able to prove a "weaker and more complicated lemma which turned out to be sufficient for all applications" matches my own experience. For instance, while working on a recent project I discovered no fewer than nine mistaken theorem statements (not just mistakes in proofs of correct theorems) in published or almost-published literature, including several by well-known experts (and two by myself). However, in all nine cases it was simple to strengthen the hypothesis or weaken the conclusion in such a way as to make the theorem true, in a way that sufficed for all the applications I know of.



          I would argue that this is because the mistaken statements were based on correct ideas, and the mistakes were simply in making those ideas precise. Or to put it differently, mathematicians get our intuitions from "well-behaved" objects: sometimes that intuition can be wrong for "pathological" objects we didn't know about, but in such cases we simply alter the definitions to exclude the pathological ones from consideration.



          On the other hand, people do sometimes get mistaken ideas. For instance, here's another quote from Voevodsky's article:




          In October 1998, Carlos Simpson ... claimed to provide an argument that implied that the main result of the "∞-groupoids" paper, which Kapranov and I had published in 1989, cannot be true. However, Kapranov and I had considered a similar critique ourselves and had convinced each other that it did not apply. I was sure that we were right until the fall of 2013 (!!).



          I can see two factors that contributed to this outrageous situation: Simpson claimed to have constructed a counterexample, but he was not able to show where the mistake was in our paper. Because of this, it was not clear whether we made a mistake somewhere in our paper or he made a mistake somewhere in his counterexample. Mathematical research currently relies on a complex system of mutual trust based on reputations. By the time Simpson’s paper appeared, both Kapranov and I had strong reputations. Simpson’s paper created doubts in our result, which led to it being unused by other researchers, but no one came forward and challenged us on it.




          In this case I do know something about the mathematics involved, and my own opinion is somewhat different from Voevodsky's. In the 2000's I was a graduate student working on higher category theory, and my impression was that in the community of higher category theory it was taken for granted that Simpson's counterexample was correct and the Kapranov-Voevodsky paper was wrong, because the claimed KV result contradicted well-known ideas in the field.



          The point here is that a community of people developing ideas together is likely to have arrived at correct intuitions, and these intuitions can flag "suspicious" results and lead to increased scrutiny of them. That is, when looking for mistaken ideas (as opposed to technical slips), it makes sense to give differing amounts of scrutiny to different claims based on whether they accord with the intuitions and expectations of experience.



          So what do you do as a student? In addition to the other good advice that's been given, I think one of your primary goals should be to train your own intuition. That way you will be better-able to evaluate whether a given result, or something like it, is probably true, before you decide whether to read and check the proof in detail.



          Of course, there is also the position that Voevodsky was led to:




          And I now do my mathematics with a proof assistant. I have a lot of wishes in terms of getting this proof assistant to work better, but at least I don’t have to go home and worry about having made a mistake in my work.




          I have a lot of respect for that position; I do plenty of formalization in proof assistants myself, and am very supportive of it. But I don't think that mathematics would be in danger of collapse without formalization, and I feel free to also do plenty of mathematics that would be prohibitively time-consuming to formalize in present-day proof assistants.







          share|cite|improve this answer














          share|cite|improve this answer



          share|cite|improve this answer








          answered 8 hours ago


























          community wiki





          Mike Shulman















          • $begingroup$
            Great answer! Upvote.
            $endgroup$
            – Ivan Di Liberti
            8 hours ago






          • 8




            $begingroup$
            Agreed; very good answer. I particularly agree that that's probably how most mistakes get caught: not by nose-to-the-grindstone checks of logical correctness, but by having sensitive mathematical noses that detect when something smells just a little bit fishy. Human intuition is so much faster than human logic. Moreover, you have to develop the inner quietude to listen to those whisperings.
            $endgroup$
            – Todd Trimble
            8 hours ago






          • 3




            $begingroup$
            +1. I would also add that sometimes published, true mathematical results do contradict the community intuition and experiences; however, in these cases, mathematicians do check the published result very carefully, in order to check correctness and to update their intuition.
            $endgroup$
            – mathworker21
            5 hours ago











          • $begingroup$
            I have some discussion of these points at the end of my blog post terrytao.wordpress.com/career-advice/… . Mathematicians with a "post-rigorous" mindset can make formal errors, but these errors tend not to propagate all the way towards collapse, because of the soundness of the intuition guiding them.
            $endgroup$
            – Terry Tao
            3 hours ago










          • $begingroup$
            The classic and beautiful book Proofs and Refutations by Imre Lakatos discusses the nature of mathematical proofs (and their iterative refinement), using Euler's polyhedron formula $V-E+F=2$ as an example.You can be wrong, but still sort-of right.
            $endgroup$
            – Fab
            3 hours ago
















          • $begingroup$
            Great answer! Upvote.
            $endgroup$
            – Ivan Di Liberti
            8 hours ago






          • 8




            $begingroup$
            Agreed; very good answer. I particularly agree that that's probably how most mistakes get caught: not by nose-to-the-grindstone checks of logical correctness, but by having sensitive mathematical noses that detect when something smells just a little bit fishy. Human intuition is so much faster than human logic. Moreover, you have to develop the inner quietude to listen to those whisperings.
            $endgroup$
            – Todd Trimble
            8 hours ago






          • 3




            $begingroup$
            +1. I would also add that sometimes published, true mathematical results do contradict the community intuition and experiences; however, in these cases, mathematicians do check the published result very carefully, in order to check correctness and to update their intuition.
            $endgroup$
            – mathworker21
            5 hours ago











          • $begingroup$
            I have some discussion of these points at the end of my blog post terrytao.wordpress.com/career-advice/… . Mathematicians with a "post-rigorous" mindset can make formal errors, but these errors tend not to propagate all the way towards collapse, because of the soundness of the intuition guiding them.
            $endgroup$
            – Terry Tao
            3 hours ago










          • $begingroup$
            The classic and beautiful book Proofs and Refutations by Imre Lakatos discusses the nature of mathematical proofs (and their iterative refinement), using Euler's polyhedron formula $V-E+F=2$ as an example.You can be wrong, but still sort-of right.
            $endgroup$
            – Fab
            3 hours ago















          $begingroup$
          Great answer! Upvote.
          $endgroup$
          – Ivan Di Liberti
          8 hours ago




          $begingroup$
          Great answer! Upvote.
          $endgroup$
          – Ivan Di Liberti
          8 hours ago




          8




          8




          $begingroup$
          Agreed; very good answer. I particularly agree that that's probably how most mistakes get caught: not by nose-to-the-grindstone checks of logical correctness, but by having sensitive mathematical noses that detect when something smells just a little bit fishy. Human intuition is so much faster than human logic. Moreover, you have to develop the inner quietude to listen to those whisperings.
          $endgroup$
          – Todd Trimble
          8 hours ago




          $begingroup$
          Agreed; very good answer. I particularly agree that that's probably how most mistakes get caught: not by nose-to-the-grindstone checks of logical correctness, but by having sensitive mathematical noses that detect when something smells just a little bit fishy. Human intuition is so much faster than human logic. Moreover, you have to develop the inner quietude to listen to those whisperings.
          $endgroup$
          – Todd Trimble
          8 hours ago




          3




          3




          $begingroup$
          +1. I would also add that sometimes published, true mathematical results do contradict the community intuition and experiences; however, in these cases, mathematicians do check the published result very carefully, in order to check correctness and to update their intuition.
          $endgroup$
          – mathworker21
          5 hours ago





          $begingroup$
          +1. I would also add that sometimes published, true mathematical results do contradict the community intuition and experiences; however, in these cases, mathematicians do check the published result very carefully, in order to check correctness and to update their intuition.
          $endgroup$
          – mathworker21
          5 hours ago













          $begingroup$
          I have some discussion of these points at the end of my blog post terrytao.wordpress.com/career-advice/… . Mathematicians with a "post-rigorous" mindset can make formal errors, but these errors tend not to propagate all the way towards collapse, because of the soundness of the intuition guiding them.
          $endgroup$
          – Terry Tao
          3 hours ago




          $begingroup$
          I have some discussion of these points at the end of my blog post terrytao.wordpress.com/career-advice/… . Mathematicians with a "post-rigorous" mindset can make formal errors, but these errors tend not to propagate all the way towards collapse, because of the soundness of the intuition guiding them.
          $endgroup$
          – Terry Tao
          3 hours ago












          $begingroup$
          The classic and beautiful book Proofs and Refutations by Imre Lakatos discusses the nature of mathematical proofs (and their iterative refinement), using Euler's polyhedron formula $V-E+F=2$ as an example.You can be wrong, but still sort-of right.
          $endgroup$
          – Fab
          3 hours ago




          $begingroup$
          The classic and beautiful book Proofs and Refutations by Imre Lakatos discusses the nature of mathematical proofs (and their iterative refinement), using Euler's polyhedron formula $V-E+F=2$ as an example.You can be wrong, but still sort-of right.
          $endgroup$
          – Fab
          3 hours ago











          25












          $begingroup$

          1. Redundancy is one big source of self-healing. A result with three different proofs is rather unlikely to be wrong. Also, people try to apply fresh results; wrong results often lead to contradictions when applied, alerting mathematicians to their wrongness. Same for proofs: Mistakes in proofs are often spotted when someone tries to adapt the proof to other questions.


          2. This is tricky. These days, using Google Scholar's "cited by" feature and various other backlink aggregators, you can get a list of papers/book that reference a given paper. Thus, if you find an error in the literature, you can track down where the "corruption" has spread. But getting corrections published is very difficult. Ted Hill and Nikolai Mnev are known for having struggled through the whole process of correcting someone else's false claims, but lots of people end up staying silent or (these days) just posting what they know somewhere on a forum like MathOverflow when someone stumbles upon the same problem. Then there are situations where no specific error can be pinpointed, but important material is simply imprecise and unreadable; fields often linger in such a limbo until someone does the thankless job of building the foundations underneath them. Katrin Wehrheim is one example of this.


          3. This question of mine got 41 votes, so yes, this is a fairly well-acknowledged problem.


          4. Ask your advisor and others. You definitely want to understand all proofs in undergraduate and lower-level graduate classes; they aren't particularly likely to be wrong, but you'll use the ideas anyway. As for advanced theory you rely upon, it depends.






          share|cite|improve this answer











          $endgroup$










          • 5




            $begingroup$
            For 4, I either agree and want to clarify or disagree slightly. You should understand all proofs presented in those classes, but you should trust the professor / the author about most of the theorems which are used in the classes but not proved, as sometimes happens when the proof of a result is too long and difficult or too far afield. It's definitely good to sometimes go beyond what is taught but in general the professor or textbook author probably decided what proofs to include based exactly on what will be useful to you in the future.
            $endgroup$
            – Will Sawin
            11 hours ago










          • $begingroup$
            Redundancy (1.) is good when the several proofs are by different authors. Several proofs in one paper, on the other hand, are not a good sign (as Arnol’d relished in pointing out).
            $endgroup$
            – Francois Ziegler
            51 mins ago
















          25












          $begingroup$

          1. Redundancy is one big source of self-healing. A result with three different proofs is rather unlikely to be wrong. Also, people try to apply fresh results; wrong results often lead to contradictions when applied, alerting mathematicians to their wrongness. Same for proofs: Mistakes in proofs are often spotted when someone tries to adapt the proof to other questions.


          2. This is tricky. These days, using Google Scholar's "cited by" feature and various other backlink aggregators, you can get a list of papers/book that reference a given paper. Thus, if you find an error in the literature, you can track down where the "corruption" has spread. But getting corrections published is very difficult. Ted Hill and Nikolai Mnev are known for having struggled through the whole process of correcting someone else's false claims, but lots of people end up staying silent or (these days) just posting what they know somewhere on a forum like MathOverflow when someone stumbles upon the same problem. Then there are situations where no specific error can be pinpointed, but important material is simply imprecise and unreadable; fields often linger in such a limbo until someone does the thankless job of building the foundations underneath them. Katrin Wehrheim is one example of this.


          3. This question of mine got 41 votes, so yes, this is a fairly well-acknowledged problem.


          4. Ask your advisor and others. You definitely want to understand all proofs in undergraduate and lower-level graduate classes; they aren't particularly likely to be wrong, but you'll use the ideas anyway. As for advanced theory you rely upon, it depends.






          share|cite|improve this answer











          $endgroup$










          • 5




            $begingroup$
            For 4, I either agree and want to clarify or disagree slightly. You should understand all proofs presented in those classes, but you should trust the professor / the author about most of the theorems which are used in the classes but not proved, as sometimes happens when the proof of a result is too long and difficult or too far afield. It's definitely good to sometimes go beyond what is taught but in general the professor or textbook author probably decided what proofs to include based exactly on what will be useful to you in the future.
            $endgroup$
            – Will Sawin
            11 hours ago










          • $begingroup$
            Redundancy (1.) is good when the several proofs are by different authors. Several proofs in one paper, on the other hand, are not a good sign (as Arnol’d relished in pointing out).
            $endgroup$
            – Francois Ziegler
            51 mins ago














          25












          25








          25





          $begingroup$

          1. Redundancy is one big source of self-healing. A result with three different proofs is rather unlikely to be wrong. Also, people try to apply fresh results; wrong results often lead to contradictions when applied, alerting mathematicians to their wrongness. Same for proofs: Mistakes in proofs are often spotted when someone tries to adapt the proof to other questions.


          2. This is tricky. These days, using Google Scholar's "cited by" feature and various other backlink aggregators, you can get a list of papers/book that reference a given paper. Thus, if you find an error in the literature, you can track down where the "corruption" has spread. But getting corrections published is very difficult. Ted Hill and Nikolai Mnev are known for having struggled through the whole process of correcting someone else's false claims, but lots of people end up staying silent or (these days) just posting what they know somewhere on a forum like MathOverflow when someone stumbles upon the same problem. Then there are situations where no specific error can be pinpointed, but important material is simply imprecise and unreadable; fields often linger in such a limbo until someone does the thankless job of building the foundations underneath them. Katrin Wehrheim is one example of this.


          3. This question of mine got 41 votes, so yes, this is a fairly well-acknowledged problem.


          4. Ask your advisor and others. You definitely want to understand all proofs in undergraduate and lower-level graduate classes; they aren't particularly likely to be wrong, but you'll use the ideas anyway. As for advanced theory you rely upon, it depends.






          share|cite|improve this answer











          $endgroup$



          1. Redundancy is one big source of self-healing. A result with three different proofs is rather unlikely to be wrong. Also, people try to apply fresh results; wrong results often lead to contradictions when applied, alerting mathematicians to their wrongness. Same for proofs: Mistakes in proofs are often spotted when someone tries to adapt the proof to other questions.


          2. This is tricky. These days, using Google Scholar's "cited by" feature and various other backlink aggregators, you can get a list of papers/book that reference a given paper. Thus, if you find an error in the literature, you can track down where the "corruption" has spread. But getting corrections published is very difficult. Ted Hill and Nikolai Mnev are known for having struggled through the whole process of correcting someone else's false claims, but lots of people end up staying silent or (these days) just posting what they know somewhere on a forum like MathOverflow when someone stumbles upon the same problem. Then there are situations where no specific error can be pinpointed, but important material is simply imprecise and unreadable; fields often linger in such a limbo until someone does the thankless job of building the foundations underneath them. Katrin Wehrheim is one example of this.


          3. This question of mine got 41 votes, so yes, this is a fairly well-acknowledged problem.


          4. Ask your advisor and others. You definitely want to understand all proofs in undergraduate and lower-level graduate classes; they aren't particularly likely to be wrong, but you'll use the ideas anyway. As for advanced theory you rely upon, it depends.







          share|cite|improve this answer














          share|cite|improve this answer



          share|cite|improve this answer








          answered 11 hours ago


























          community wiki





          darij grinberg











          • 5




            $begingroup$
            For 4, I either agree and want to clarify or disagree slightly. You should understand all proofs presented in those classes, but you should trust the professor / the author about most of the theorems which are used in the classes but not proved, as sometimes happens when the proof of a result is too long and difficult or too far afield. It's definitely good to sometimes go beyond what is taught but in general the professor or textbook author probably decided what proofs to include based exactly on what will be useful to you in the future.
            $endgroup$
            – Will Sawin
            11 hours ago










          • $begingroup$
            Redundancy (1.) is good when the several proofs are by different authors. Several proofs in one paper, on the other hand, are not a good sign (as Arnol’d relished in pointing out).
            $endgroup$
            – Francois Ziegler
            51 mins ago













          • 5




            $begingroup$
            For 4, I either agree and want to clarify or disagree slightly. You should understand all proofs presented in those classes, but you should trust the professor / the author about most of the theorems which are used in the classes but not proved, as sometimes happens when the proof of a result is too long and difficult or too far afield. It's definitely good to sometimes go beyond what is taught but in general the professor or textbook author probably decided what proofs to include based exactly on what will be useful to you in the future.
            $endgroup$
            – Will Sawin
            11 hours ago










          • $begingroup$
            Redundancy (1.) is good when the several proofs are by different authors. Several proofs in one paper, on the other hand, are not a good sign (as Arnol’d relished in pointing out).
            $endgroup$
            – Francois Ziegler
            51 mins ago








          5




          5




          $begingroup$
          For 4, I either agree and want to clarify or disagree slightly. You should understand all proofs presented in those classes, but you should trust the professor / the author about most of the theorems which are used in the classes but not proved, as sometimes happens when the proof of a result is too long and difficult or too far afield. It's definitely good to sometimes go beyond what is taught but in general the professor or textbook author probably decided what proofs to include based exactly on what will be useful to you in the future.
          $endgroup$
          – Will Sawin
          11 hours ago




          $begingroup$
          For 4, I either agree and want to clarify or disagree slightly. You should understand all proofs presented in those classes, but you should trust the professor / the author about most of the theorems which are used in the classes but not proved, as sometimes happens when the proof of a result is too long and difficult or too far afield. It's definitely good to sometimes go beyond what is taught but in general the professor or textbook author probably decided what proofs to include based exactly on what will be useful to you in the future.
          $endgroup$
          – Will Sawin
          11 hours ago












          $begingroup$
          Redundancy (1.) is good when the several proofs are by different authors. Several proofs in one paper, on the other hand, are not a good sign (as Arnol’d relished in pointing out).
          $endgroup$
          – Francois Ziegler
          51 mins ago





          $begingroup$
          Redundancy (1.) is good when the several proofs are by different authors. Several proofs in one paper, on the other hand, are not a good sign (as Arnol’d relished in pointing out).
          $endgroup$
          – Francois Ziegler
          51 mins ago












          16












          $begingroup$

          This is a broad question, but you may find it helpful to read The Existential Risk of Math Errors. It suggests a certain robustness of the mathematical edifice, which I actually think extends to the natural sciences as a whole. (Newtonian mechanics is "wrong" in a fundamental sense, but neither the development of relativistic mechanics nor the discovery of quantum mechanics has caused the collapse of classical mechanics.)



          This quote in particular from Gian-Carlo Rota bears on your points 1 and 2:




          When the Germans were planning to publish Hilbert’s collected papers
          and to present him with a set on the occasion of one of his later
          birthdays, they realized that they could not publish the papers in
          their original versions because they were full of errors, some of them
          quite serious. Thereupon they hired a young unemployed mathematician,
          Olga Taussky-Todd, to go over Hilbert’s papers and correct all
          mistakes. Olga labored for three years; it turned out that all
          mistakes could be corrected without any major changes in the statement
          of the theorems. There was one exception, a paper Hilbert wrote in his
          old age, which could not be fixed; it was a purported proof of the
          continuum hypothesis, you will find it in a volume of the
          Mathematische Annalen of the early thirties. At last, on Hilbert’s
          birthday, a freshly printed set of Hilbert’s collected papers was
          presented to the Geheimrat. Hilbert leafed through them carefully and
          did not notice anything.







          share|cite|improve this answer











          $endgroup$










          • 8




            $begingroup$
            Rota is unreliable here: Hilbert’s published theorems sometimes were incorrect as stated. See McLarty on the note in the Annalen about his famous result on invariants. webusers.imj-prg.fr/~michael.harris/theology.pdf
            $endgroup$
            – Matt F.
            10 hours ago







          • 11




            $begingroup$
            Rota is a great story-teller, but generally speaking people should fact-check him on his pronouncements.
            $endgroup$
            – Todd Trimble
            8 hours ago






          • 6




            $begingroup$
            @MattF: The word "major" in "major changes" is doing a lot of work, I suspect. An author's close collaborators usually have a good idea of what the author was meaning to say, and thus can fix all sorts of imprecisions that an outsider would be stumped by.
            $endgroup$
            – darij grinberg
            4 hours ago
















          16












          $begingroup$

          This is a broad question, but you may find it helpful to read The Existential Risk of Math Errors. It suggests a certain robustness of the mathematical edifice, which I actually think extends to the natural sciences as a whole. (Newtonian mechanics is "wrong" in a fundamental sense, but neither the development of relativistic mechanics nor the discovery of quantum mechanics has caused the collapse of classical mechanics.)



          This quote in particular from Gian-Carlo Rota bears on your points 1 and 2:




          When the Germans were planning to publish Hilbert’s collected papers
          and to present him with a set on the occasion of one of his later
          birthdays, they realized that they could not publish the papers in
          their original versions because they were full of errors, some of them
          quite serious. Thereupon they hired a young unemployed mathematician,
          Olga Taussky-Todd, to go over Hilbert’s papers and correct all
          mistakes. Olga labored for three years; it turned out that all
          mistakes could be corrected without any major changes in the statement
          of the theorems. There was one exception, a paper Hilbert wrote in his
          old age, which could not be fixed; it was a purported proof of the
          continuum hypothesis, you will find it in a volume of the
          Mathematische Annalen of the early thirties. At last, on Hilbert’s
          birthday, a freshly printed set of Hilbert’s collected papers was
          presented to the Geheimrat. Hilbert leafed through them carefully and
          did not notice anything.







          share|cite|improve this answer











          $endgroup$










          • 8




            $begingroup$
            Rota is unreliable here: Hilbert’s published theorems sometimes were incorrect as stated. See McLarty on the note in the Annalen about his famous result on invariants. webusers.imj-prg.fr/~michael.harris/theology.pdf
            $endgroup$
            – Matt F.
            10 hours ago







          • 11




            $begingroup$
            Rota is a great story-teller, but generally speaking people should fact-check him on his pronouncements.
            $endgroup$
            – Todd Trimble
            8 hours ago






          • 6




            $begingroup$
            @MattF: The word "major" in "major changes" is doing a lot of work, I suspect. An author's close collaborators usually have a good idea of what the author was meaning to say, and thus can fix all sorts of imprecisions that an outsider would be stumped by.
            $endgroup$
            – darij grinberg
            4 hours ago














          16












          16








          16





          $begingroup$

          This is a broad question, but you may find it helpful to read The Existential Risk of Math Errors. It suggests a certain robustness of the mathematical edifice, which I actually think extends to the natural sciences as a whole. (Newtonian mechanics is "wrong" in a fundamental sense, but neither the development of relativistic mechanics nor the discovery of quantum mechanics has caused the collapse of classical mechanics.)



          This quote in particular from Gian-Carlo Rota bears on your points 1 and 2:




          When the Germans were planning to publish Hilbert’s collected papers
          and to present him with a set on the occasion of one of his later
          birthdays, they realized that they could not publish the papers in
          their original versions because they were full of errors, some of them
          quite serious. Thereupon they hired a young unemployed mathematician,
          Olga Taussky-Todd, to go over Hilbert’s papers and correct all
          mistakes. Olga labored for three years; it turned out that all
          mistakes could be corrected without any major changes in the statement
          of the theorems. There was one exception, a paper Hilbert wrote in his
          old age, which could not be fixed; it was a purported proof of the
          continuum hypothesis, you will find it in a volume of the
          Mathematische Annalen of the early thirties. At last, on Hilbert’s
          birthday, a freshly printed set of Hilbert’s collected papers was
          presented to the Geheimrat. Hilbert leafed through them carefully and
          did not notice anything.







          share|cite|improve this answer











          $endgroup$



          This is a broad question, but you may find it helpful to read The Existential Risk of Math Errors. It suggests a certain robustness of the mathematical edifice, which I actually think extends to the natural sciences as a whole. (Newtonian mechanics is "wrong" in a fundamental sense, but neither the development of relativistic mechanics nor the discovery of quantum mechanics has caused the collapse of classical mechanics.)



          This quote in particular from Gian-Carlo Rota bears on your points 1 and 2:




          When the Germans were planning to publish Hilbert’s collected papers
          and to present him with a set on the occasion of one of his later
          birthdays, they realized that they could not publish the papers in
          their original versions because they were full of errors, some of them
          quite serious. Thereupon they hired a young unemployed mathematician,
          Olga Taussky-Todd, to go over Hilbert’s papers and correct all
          mistakes. Olga labored for three years; it turned out that all
          mistakes could be corrected without any major changes in the statement
          of the theorems. There was one exception, a paper Hilbert wrote in his
          old age, which could not be fixed; it was a purported proof of the
          continuum hypothesis, you will find it in a volume of the
          Mathematische Annalen of the early thirties. At last, on Hilbert’s
          birthday, a freshly printed set of Hilbert’s collected papers was
          presented to the Geheimrat. Hilbert leafed through them carefully and
          did not notice anything.








          share|cite|improve this answer














          share|cite|improve this answer



          share|cite|improve this answer








          edited 4 hours ago


























          community wiki





          4 revs, 2 users 98%
          Carlo Beenakker











          • 8




            $begingroup$
            Rota is unreliable here: Hilbert’s published theorems sometimes were incorrect as stated. See McLarty on the note in the Annalen about his famous result on invariants. webusers.imj-prg.fr/~michael.harris/theology.pdf
            $endgroup$
            – Matt F.
            10 hours ago







          • 11




            $begingroup$
            Rota is a great story-teller, but generally speaking people should fact-check him on his pronouncements.
            $endgroup$
            – Todd Trimble
            8 hours ago






          • 6




            $begingroup$
            @MattF: The word "major" in "major changes" is doing a lot of work, I suspect. An author's close collaborators usually have a good idea of what the author was meaning to say, and thus can fix all sorts of imprecisions that an outsider would be stumped by.
            $endgroup$
            – darij grinberg
            4 hours ago













          • 8




            $begingroup$
            Rota is unreliable here: Hilbert’s published theorems sometimes were incorrect as stated. See McLarty on the note in the Annalen about his famous result on invariants. webusers.imj-prg.fr/~michael.harris/theology.pdf
            $endgroup$
            – Matt F.
            10 hours ago







          • 11




            $begingroup$
            Rota is a great story-teller, but generally speaking people should fact-check him on his pronouncements.
            $endgroup$
            – Todd Trimble
            8 hours ago






          • 6




            $begingroup$
            @MattF: The word "major" in "major changes" is doing a lot of work, I suspect. An author's close collaborators usually have a good idea of what the author was meaning to say, and thus can fix all sorts of imprecisions that an outsider would be stumped by.
            $endgroup$
            – darij grinberg
            4 hours ago








          8




          8




          $begingroup$
          Rota is unreliable here: Hilbert’s published theorems sometimes were incorrect as stated. See McLarty on the note in the Annalen about his famous result on invariants. webusers.imj-prg.fr/~michael.harris/theology.pdf
          $endgroup$
          – Matt F.
          10 hours ago





          $begingroup$
          Rota is unreliable here: Hilbert’s published theorems sometimes were incorrect as stated. See McLarty on the note in the Annalen about his famous result on invariants. webusers.imj-prg.fr/~michael.harris/theology.pdf
          $endgroup$
          – Matt F.
          10 hours ago





          11




          11




          $begingroup$
          Rota is a great story-teller, but generally speaking people should fact-check him on his pronouncements.
          $endgroup$
          – Todd Trimble
          8 hours ago




          $begingroup$
          Rota is a great story-teller, but generally speaking people should fact-check him on his pronouncements.
          $endgroup$
          – Todd Trimble
          8 hours ago




          6




          6




          $begingroup$
          @MattF: The word "major" in "major changes" is doing a lot of work, I suspect. An author's close collaborators usually have a good idea of what the author was meaning to say, and thus can fix all sorts of imprecisions that an outsider would be stumped by.
          $endgroup$
          – darij grinberg
          4 hours ago





          $begingroup$
          @MattF: The word "major" in "major changes" is doing a lot of work, I suspect. An author's close collaborators usually have a good idea of what the author was meaning to say, and thus can fix all sorts of imprecisions that an outsider would be stumped by.
          $endgroup$
          – darij grinberg
          4 hours ago












          11












          $begingroup$

          If a result is not used much, then its veracity does not matter much for the rest of mathematics.



          Otherwise, there may be several proofs of the result, which makes it much more probable (in a general sense) that the result is true. Importantly, there is usually some explanation or understanding of why the result is true, that is, the ideas behind the result.



          Also, people try and do construct counterexamples to disprove a result, if they do not see why the result should be true.






          share|cite|improve this answer











          $endgroup$



















            11












            $begingroup$

            If a result is not used much, then its veracity does not matter much for the rest of mathematics.



            Otherwise, there may be several proofs of the result, which makes it much more probable (in a general sense) that the result is true. Importantly, there is usually some explanation or understanding of why the result is true, that is, the ideas behind the result.



            Also, people try and do construct counterexamples to disprove a result, if they do not see why the result should be true.






            share|cite|improve this answer











            $endgroup$

















              11












              11








              11





              $begingroup$

              If a result is not used much, then its veracity does not matter much for the rest of mathematics.



              Otherwise, there may be several proofs of the result, which makes it much more probable (in a general sense) that the result is true. Importantly, there is usually some explanation or understanding of why the result is true, that is, the ideas behind the result.



              Also, people try and do construct counterexamples to disprove a result, if they do not see why the result should be true.






              share|cite|improve this answer











              $endgroup$



              If a result is not used much, then its veracity does not matter much for the rest of mathematics.



              Otherwise, there may be several proofs of the result, which makes it much more probable (in a general sense) that the result is true. Importantly, there is usually some explanation or understanding of why the result is true, that is, the ideas behind the result.



              Also, people try and do construct counterexamples to disprove a result, if they do not see why the result should be true.







              share|cite|improve this answer














              share|cite|improve this answer



              share|cite|improve this answer








              edited 9 hours ago


























              community wiki





              2 revs
              Iosif Pinelis

























                  11












                  $begingroup$

                  It might feel like the chain of citations goes ad infinitum,
                  but of course this can't literally be the case. Indeed as long as
                  mathematics is (at least primarily) an endeavor by and for humans,
                  the entire chain of reasoning must be graspable by about the time a mathematician
                  reaches PhD level. Much of it will not yet be accessible to an advanced
                  college student such as the OP, and modern mathematics has gone in
                  so many directions that no one mathematician can grasp more than
                  a small sliver of the frontier $-$ which is why modern research mathematics
                  requires specialization, and a mathematical community large enough to support
                  such a large frontier. Inevitably errors occur, and a few of those
                  propagate for some time before being caught. But the enterprise as a whole
                  is self-correcting (as already explained in several ways in other answers).






                  share|cite|improve this answer











                  $endgroup$



















                    11












                    $begingroup$

                    It might feel like the chain of citations goes ad infinitum,
                    but of course this can't literally be the case. Indeed as long as
                    mathematics is (at least primarily) an endeavor by and for humans,
                    the entire chain of reasoning must be graspable by about the time a mathematician
                    reaches PhD level. Much of it will not yet be accessible to an advanced
                    college student such as the OP, and modern mathematics has gone in
                    so many directions that no one mathematician can grasp more than
                    a small sliver of the frontier $-$ which is why modern research mathematics
                    requires specialization, and a mathematical community large enough to support
                    such a large frontier. Inevitably errors occur, and a few of those
                    propagate for some time before being caught. But the enterprise as a whole
                    is self-correcting (as already explained in several ways in other answers).






                    share|cite|improve this answer











                    $endgroup$

















                      11












                      11








                      11





                      $begingroup$

                      It might feel like the chain of citations goes ad infinitum,
                      but of course this can't literally be the case. Indeed as long as
                      mathematics is (at least primarily) an endeavor by and for humans,
                      the entire chain of reasoning must be graspable by about the time a mathematician
                      reaches PhD level. Much of it will not yet be accessible to an advanced
                      college student such as the OP, and modern mathematics has gone in
                      so many directions that no one mathematician can grasp more than
                      a small sliver of the frontier $-$ which is why modern research mathematics
                      requires specialization, and a mathematical community large enough to support
                      such a large frontier. Inevitably errors occur, and a few of those
                      propagate for some time before being caught. But the enterprise as a whole
                      is self-correcting (as already explained in several ways in other answers).






                      share|cite|improve this answer











                      $endgroup$



                      It might feel like the chain of citations goes ad infinitum,
                      but of course this can't literally be the case. Indeed as long as
                      mathematics is (at least primarily) an endeavor by and for humans,
                      the entire chain of reasoning must be graspable by about the time a mathematician
                      reaches PhD level. Much of it will not yet be accessible to an advanced
                      college student such as the OP, and modern mathematics has gone in
                      so many directions that no one mathematician can grasp more than
                      a small sliver of the frontier $-$ which is why modern research mathematics
                      requires specialization, and a mathematical community large enough to support
                      such a large frontier. Inevitably errors occur, and a few of those
                      propagate for some time before being caught. But the enterprise as a whole
                      is self-correcting (as already explained in several ways in other answers).







                      share|cite|improve this answer














                      share|cite|improve this answer



                      share|cite|improve this answer








                      answered 6 hours ago


























                      community wiki





                      Noam D. Elkies

























                          5












                          $begingroup$

                          While the responses to this question have so far have generally (and correctly) focused on the robust nature of the edifice of mathematics as a whole, it might be worth pointing out by way of contrast that sometimes there are indeed doubts about foundational issues within a specific subfield (because, e.g., an "important" paper is known to have flaws), and that can be extremely deleterious to the field in question. Something along these lines is discussed in this Quanta magazine article: https://www.quantamagazine.org/the-fight-to-fix-symplectic-geometry-20170209/. (And I would say that what is described there is not even the most extreme example of what can happen.)






                          share|cite|improve this answer











                          $endgroup$










                          • 1




                            $begingroup$
                            (This case is also mentioned by @darijgrinberg.)
                            $endgroup$
                            – Francois Ziegler
                            1 hour ago
















                          5












                          $begingroup$

                          While the responses to this question have so far have generally (and correctly) focused on the robust nature of the edifice of mathematics as a whole, it might be worth pointing out by way of contrast that sometimes there are indeed doubts about foundational issues within a specific subfield (because, e.g., an "important" paper is known to have flaws), and that can be extremely deleterious to the field in question. Something along these lines is discussed in this Quanta magazine article: https://www.quantamagazine.org/the-fight-to-fix-symplectic-geometry-20170209/. (And I would say that what is described there is not even the most extreme example of what can happen.)






                          share|cite|improve this answer











                          $endgroup$










                          • 1




                            $begingroup$
                            (This case is also mentioned by @darijgrinberg.)
                            $endgroup$
                            – Francois Ziegler
                            1 hour ago














                          5












                          5








                          5





                          $begingroup$

                          While the responses to this question have so far have generally (and correctly) focused on the robust nature of the edifice of mathematics as a whole, it might be worth pointing out by way of contrast that sometimes there are indeed doubts about foundational issues within a specific subfield (because, e.g., an "important" paper is known to have flaws), and that can be extremely deleterious to the field in question. Something along these lines is discussed in this Quanta magazine article: https://www.quantamagazine.org/the-fight-to-fix-symplectic-geometry-20170209/. (And I would say that what is described there is not even the most extreme example of what can happen.)






                          share|cite|improve this answer











                          $endgroup$



                          While the responses to this question have so far have generally (and correctly) focused on the robust nature of the edifice of mathematics as a whole, it might be worth pointing out by way of contrast that sometimes there are indeed doubts about foundational issues within a specific subfield (because, e.g., an "important" paper is known to have flaws), and that can be extremely deleterious to the field in question. Something along these lines is discussed in this Quanta magazine article: https://www.quantamagazine.org/the-fight-to-fix-symplectic-geometry-20170209/. (And I would say that what is described there is not even the most extreme example of what can happen.)







                          share|cite|improve this answer














                          share|cite|improve this answer



                          share|cite|improve this answer








                          answered 2 hours ago


























                          community wiki





                          Sam Hopkins











                          • 1




                            $begingroup$
                            (This case is also mentioned by @darijgrinberg.)
                            $endgroup$
                            – Francois Ziegler
                            1 hour ago













                          • 1




                            $begingroup$
                            (This case is also mentioned by @darijgrinberg.)
                            $endgroup$
                            – Francois Ziegler
                            1 hour ago








                          1




                          1




                          $begingroup$
                          (This case is also mentioned by @darijgrinberg.)
                          $endgroup$
                          – Francois Ziegler
                          1 hour ago





                          $begingroup$
                          (This case is also mentioned by @darijgrinberg.)
                          $endgroup$
                          – Francois Ziegler
                          1 hour ago












                          2












                          $begingroup$

                          Further to M. Shulman’s advice to “develop your intuition”, it’s probably worth adding that this is often done by understanding many examples, special cases whose moving parts are already transparent to you. You get something simpler and more robust. And psychologically at least, “trust” in a result often relies on familiarity with a library of such cases, more than on line-checking a general proof. (I would also guess that many innocuous “errors” only reflect some overshoot in generality when streamlining things for publication, and that is why not everything collapses.) Or, in the polemical words of Arnol’d (2004):




                          There are two principal ways to formulate mathematical assertions (problems, conjectures, theorems,...): Russian and French. The Russian way is to choose the most simple and specific case (so that nobody could simplify the formulation preserving the main point). The French way is to generalize the statement as far as nobody could generalize it further.




                          (That’s not to say that “top-down” never wins — e.g. reputedly, only the discovery of Bott periodicity settled a “spirited controversy” on specific homotopy groups (1959, p. 355 and Math review).)






                          share|cite|improve this answer











                          $endgroup$



















                            2












                            $begingroup$

                            Further to M. Shulman’s advice to “develop your intuition”, it’s probably worth adding that this is often done by understanding many examples, special cases whose moving parts are already transparent to you. You get something simpler and more robust. And psychologically at least, “trust” in a result often relies on familiarity with a library of such cases, more than on line-checking a general proof. (I would also guess that many innocuous “errors” only reflect some overshoot in generality when streamlining things for publication, and that is why not everything collapses.) Or, in the polemical words of Arnol’d (2004):




                            There are two principal ways to formulate mathematical assertions (problems, conjectures, theorems,...): Russian and French. The Russian way is to choose the most simple and specific case (so that nobody could simplify the formulation preserving the main point). The French way is to generalize the statement as far as nobody could generalize it further.




                            (That’s not to say that “top-down” never wins — e.g. reputedly, only the discovery of Bott periodicity settled a “spirited controversy” on specific homotopy groups (1959, p. 355 and Math review).)






                            share|cite|improve this answer











                            $endgroup$

















                              2












                              2








                              2





                              $begingroup$

                              Further to M. Shulman’s advice to “develop your intuition”, it’s probably worth adding that this is often done by understanding many examples, special cases whose moving parts are already transparent to you. You get something simpler and more robust. And psychologically at least, “trust” in a result often relies on familiarity with a library of such cases, more than on line-checking a general proof. (I would also guess that many innocuous “errors” only reflect some overshoot in generality when streamlining things for publication, and that is why not everything collapses.) Or, in the polemical words of Arnol’d (2004):




                              There are two principal ways to formulate mathematical assertions (problems, conjectures, theorems,...): Russian and French. The Russian way is to choose the most simple and specific case (so that nobody could simplify the formulation preserving the main point). The French way is to generalize the statement as far as nobody could generalize it further.




                              (That’s not to say that “top-down” never wins — e.g. reputedly, only the discovery of Bott periodicity settled a “spirited controversy” on specific homotopy groups (1959, p. 355 and Math review).)






                              share|cite|improve this answer











                              $endgroup$



                              Further to M. Shulman’s advice to “develop your intuition”, it’s probably worth adding that this is often done by understanding many examples, special cases whose moving parts are already transparent to you. You get something simpler and more robust. And psychologically at least, “trust” in a result often relies on familiarity with a library of such cases, more than on line-checking a general proof. (I would also guess that many innocuous “errors” only reflect some overshoot in generality when streamlining things for publication, and that is why not everything collapses.) Or, in the polemical words of Arnol’d (2004):




                              There are two principal ways to formulate mathematical assertions (problems, conjectures, theorems,...): Russian and French. The Russian way is to choose the most simple and specific case (so that nobody could simplify the formulation preserving the main point). The French way is to generalize the statement as far as nobody could generalize it further.




                              (That’s not to say that “top-down” never wins — e.g. reputedly, only the discovery of Bott periodicity settled a “spirited controversy” on specific homotopy groups (1959, p. 355 and Math review).)







                              share|cite|improve this answer














                              share|cite|improve this answer



                              share|cite|improve this answer








                              edited 2 hours ago


























                              community wiki





                              2 revs
                              Francois Ziegler

























                                  0












                                  $begingroup$

                                  Well, because of a multitude of reasons, including:



                                  1. Mathematicians rarely make mistakes in simple proofs. And a lot of the basics of math are simple theorems that follow easily from the formal definitions.


                                  2. Mistakes get corrected quickly before claims get accepted and used:



                                    2.1 Most people double-check and re-check their own proofs before claiming to have proved something significant (although some don't - I've heard a rumor that Saharon Shelah had a bunch of papers with errors and brushed this off by saying they could be be easily rectified)



                                    2.2 Mathematicians are a community and check each other's work - so there's much less chance of mistakes escaping scrutiny.



                                  3. Doubt: People - mathematicians and users of math - don't just accept new claims at face value. Even if they don't have the time to check proofs themselves, they'll treat new claims/results as somewhat suspect until sufficient (or apparently sufficient) verification happens.


                                  4. Using an erroneous theorem usually leads to obvious problems, so whoever adopts an erroneous theorem as valid typically falls flat on their face, and this easily leads to doubt being cast on their assumptions - namely, the faulty theorem.



                                  5. New mistakes don't invalidate past work. If a field of mathematics has some valid, no-mistakes basis, then our mistakes building on that basis don't invalidate it. At most, we may get confused and doubt some of it - but that will only make us double-check its validity or look for a counter-example - which will fail.





                                  share|cite|improve this answer











                                  $endgroup$



















                                    0












                                    $begingroup$

                                    Well, because of a multitude of reasons, including:



                                    1. Mathematicians rarely make mistakes in simple proofs. And a lot of the basics of math are simple theorems that follow easily from the formal definitions.


                                    2. Mistakes get corrected quickly before claims get accepted and used:



                                      2.1 Most people double-check and re-check their own proofs before claiming to have proved something significant (although some don't - I've heard a rumor that Saharon Shelah had a bunch of papers with errors and brushed this off by saying they could be be easily rectified)



                                      2.2 Mathematicians are a community and check each other's work - so there's much less chance of mistakes escaping scrutiny.



                                    3. Doubt: People - mathematicians and users of math - don't just accept new claims at face value. Even if they don't have the time to check proofs themselves, they'll treat new claims/results as somewhat suspect until sufficient (or apparently sufficient) verification happens.


                                    4. Using an erroneous theorem usually leads to obvious problems, so whoever adopts an erroneous theorem as valid typically falls flat on their face, and this easily leads to doubt being cast on their assumptions - namely, the faulty theorem.



                                    5. New mistakes don't invalidate past work. If a field of mathematics has some valid, no-mistakes basis, then our mistakes building on that basis don't invalidate it. At most, we may get confused and doubt some of it - but that will only make us double-check its validity or look for a counter-example - which will fail.





                                    share|cite|improve this answer











                                    $endgroup$

















                                      0












                                      0








                                      0





                                      $begingroup$

                                      Well, because of a multitude of reasons, including:



                                      1. Mathematicians rarely make mistakes in simple proofs. And a lot of the basics of math are simple theorems that follow easily from the formal definitions.


                                      2. Mistakes get corrected quickly before claims get accepted and used:



                                        2.1 Most people double-check and re-check their own proofs before claiming to have proved something significant (although some don't - I've heard a rumor that Saharon Shelah had a bunch of papers with errors and brushed this off by saying they could be be easily rectified)



                                        2.2 Mathematicians are a community and check each other's work - so there's much less chance of mistakes escaping scrutiny.



                                      3. Doubt: People - mathematicians and users of math - don't just accept new claims at face value. Even if they don't have the time to check proofs themselves, they'll treat new claims/results as somewhat suspect until sufficient (or apparently sufficient) verification happens.


                                      4. Using an erroneous theorem usually leads to obvious problems, so whoever adopts an erroneous theorem as valid typically falls flat on their face, and this easily leads to doubt being cast on their assumptions - namely, the faulty theorem.



                                      5. New mistakes don't invalidate past work. If a field of mathematics has some valid, no-mistakes basis, then our mistakes building on that basis don't invalidate it. At most, we may get confused and doubt some of it - but that will only make us double-check its validity or look for a counter-example - which will fail.





                                      share|cite|improve this answer











                                      $endgroup$



                                      Well, because of a multitude of reasons, including:



                                      1. Mathematicians rarely make mistakes in simple proofs. And a lot of the basics of math are simple theorems that follow easily from the formal definitions.


                                      2. Mistakes get corrected quickly before claims get accepted and used:



                                        2.1 Most people double-check and re-check their own proofs before claiming to have proved something significant (although some don't - I've heard a rumor that Saharon Shelah had a bunch of papers with errors and brushed this off by saying they could be be easily rectified)



                                        2.2 Mathematicians are a community and check each other's work - so there's much less chance of mistakes escaping scrutiny.



                                      3. Doubt: People - mathematicians and users of math - don't just accept new claims at face value. Even if they don't have the time to check proofs themselves, they'll treat new claims/results as somewhat suspect until sufficient (or apparently sufficient) verification happens.


                                      4. Using an erroneous theorem usually leads to obvious problems, so whoever adopts an erroneous theorem as valid typically falls flat on their face, and this easily leads to doubt being cast on their assumptions - namely, the faulty theorem.



                                      5. New mistakes don't invalidate past work. If a field of mathematics has some valid, no-mistakes basis, then our mistakes building on that basis don't invalidate it. At most, we may get confused and doubt some of it - but that will only make us double-check its validity or look for a counter-example - which will fail.






                                      share|cite|improve this answer














                                      share|cite|improve this answer



                                      share|cite|improve this answer








                                      answered 2 hours ago


























                                      community wiki





                                      einpoklum

























                                          -1












                                          $begingroup$

                                          Sometimes these collapses happen. Ole Peters and Murray Gell-Mann have a 2016 paper about a mistake in Expected Utility Theory, which they track down to a mistake in a Bernoulli paper from 1738.



                                          I'm an outsider looking in, but it sure looks like much of that entire field is in serious trouble. There's a post on this here: https://ergodicityeconomics.com/2018/02/16/the-trouble-with-bernoulli-1738/






                                          share|cite|improve this answer











                                          $endgroup$














                                          • $begingroup$
                                            While I know almost nothing about economics, my impression is that this critique is not accepted by mainstream economists.
                                            $endgroup$
                                            – Andy Putman
                                            2 hours ago















                                          -1












                                          $begingroup$

                                          Sometimes these collapses happen. Ole Peters and Murray Gell-Mann have a 2016 paper about a mistake in Expected Utility Theory, which they track down to a mistake in a Bernoulli paper from 1738.



                                          I'm an outsider looking in, but it sure looks like much of that entire field is in serious trouble. There's a post on this here: https://ergodicityeconomics.com/2018/02/16/the-trouble-with-bernoulli-1738/






                                          share|cite|improve this answer











                                          $endgroup$














                                          • $begingroup$
                                            While I know almost nothing about economics, my impression is that this critique is not accepted by mainstream economists.
                                            $endgroup$
                                            – Andy Putman
                                            2 hours ago













                                          -1












                                          -1








                                          -1





                                          $begingroup$

                                          Sometimes these collapses happen. Ole Peters and Murray Gell-Mann have a 2016 paper about a mistake in Expected Utility Theory, which they track down to a mistake in a Bernoulli paper from 1738.



                                          I'm an outsider looking in, but it sure looks like much of that entire field is in serious trouble. There's a post on this here: https://ergodicityeconomics.com/2018/02/16/the-trouble-with-bernoulli-1738/






                                          share|cite|improve this answer











                                          $endgroup$



                                          Sometimes these collapses happen. Ole Peters and Murray Gell-Mann have a 2016 paper about a mistake in Expected Utility Theory, which they track down to a mistake in a Bernoulli paper from 1738.



                                          I'm an outsider looking in, but it sure looks like much of that entire field is in serious trouble. There's a post on this here: https://ergodicityeconomics.com/2018/02/16/the-trouble-with-bernoulli-1738/







                                          share|cite|improve this answer














                                          share|cite|improve this answer



                                          share|cite|improve this answer








                                          answered 3 hours ago


























                                          community wiki





                                          larspars















                                          • $begingroup$
                                            While I know almost nothing about economics, my impression is that this critique is not accepted by mainstream economists.
                                            $endgroup$
                                            – Andy Putman
                                            2 hours ago
















                                          • $begingroup$
                                            While I know almost nothing about economics, my impression is that this critique is not accepted by mainstream economists.
                                            $endgroup$
                                            – Andy Putman
                                            2 hours ago















                                          $begingroup$
                                          While I know almost nothing about economics, my impression is that this critique is not accepted by mainstream economists.
                                          $endgroup$
                                          – Andy Putman
                                          2 hours ago




                                          $begingroup$
                                          While I know almost nothing about economics, my impression is that this critique is not accepted by mainstream economists.
                                          $endgroup$
                                          – Andy Putman
                                          2 hours ago

















                                          draft saved

                                          draft discarded
















































                                          Thanks for contributing an answer to MathOverflow!


                                          • Please be sure to answer the question. Provide details and share your research!

                                          But avoid


                                          • Asking for help, clarification, or responding to other answers.

                                          • Making statements based on opinion; back them up with references or personal experience.

                                          Use MathJax to format equations. MathJax reference.


                                          To learn more, see our tips on writing great answers.




                                          draft saved


                                          draft discarded














                                          StackExchange.ready(
                                          function ()
                                          StackExchange.openid.initPostLogin('.new-post-login', 'https%3a%2f%2fmathoverflow.net%2fquestions%2f338607%2fwhy-doesnt-mathematics-collapse-down-even-though-humans-quite-often-make-mista%23new-answer', 'question_page');

                                          );

                                          Post as a guest















                                          Required, but never shown





















































                                          Required, but never shown














                                          Required, but never shown












                                          Required, but never shown







                                          Required, but never shown

































                                          Required, but never shown














                                          Required, but never shown












                                          Required, but never shown







                                          Required, but never shown







                                          Popular posts from this blog

                                          Invision Community Contents History See also References External links Navigation menuProprietaryinvisioncommunity.comIPS Community ForumsIPS Community Forumsthis blog entry"License Changes, IP.Board 3.4, and the Future""Interview -- Matt Mecham of Ibforums""CEO Invision Power Board, Matt Mecham Is a Liar, Thief!"IPB License Explanation 1.3, 1.3.1, 2.0, and 2.1ArchivedSecurity Fixes, Updates And Enhancements For IPB 1.3.1Archived"New Demo Accounts - Invision Power Services"the original"New Default Skin"the original"Invision Power Board 3.0.0 and Applications Released"the original"Archived copy"the original"Perpetual licenses being done away with""Release Notes - Invision Power Services""Introducing: IPS Community Suite 4!"Invision Community Release Notes

                                          Canceling a color specificationRandomly assigning color to Graphics3D objects?Default color for Filling in Mathematica 9Coloring specific elements of sets with a prime modified order in an array plotHow to pick a color differing significantly from the colors already in a given color list?Detection of the text colorColor numbers based on their valueCan color schemes for use with ColorData include opacity specification?My dynamic color schemes

                                          Tom Holland Mục lục Đầu đời và giáo dục | Sự nghiệp | Cuộc sống cá nhân | Phim tham gia | Giải thưởng và đề cử | Chú thích | Liên kết ngoài | Trình đơn chuyển hướngProfile“Person Details for Thomas Stanley Holland, "England and Wales Birth Registration Index, 1837-2008" — FamilySearch.org”"Meet Tom Holland... the 16-year-old star of The Impossible""Schoolboy actor Tom Holland finds himself in Oscar contention for role in tsunami drama"“Naomi Watts on the Prince William and Harry's reaction to her film about the late Princess Diana”lưu trữ"Holland and Pflueger Are West End's Two New 'Billy Elliots'""I'm so envious of my son, the movie star! British writer Dominic Holland's spent 20 years trying to crack Hollywood - but he's been beaten to it by a very unlikely rival"“Richard and Margaret Povey of Jersey, Channel Islands, UK: Information about Thomas Stanley Holland”"Tom Holland to play Billy Elliot""New Billy Elliot leaving the garage"Billy Elliot the Musical - Tom Holland - Billy"A Tale of four Billys: Tom Holland""The Feel Good Factor""Thames Christian College schoolboys join Myleene Klass for The Feelgood Factor""Government launches £600,000 arts bursaries pilot""BILLY's Chapman, Holland, Gardner & Jackson-Keen Visit Prime Minister""Elton John 'blown away' by Billy Elliot fifth birthday" (video with John's interview and fragments of Holland's performance)"First News interviews Arrietty's Tom Holland"“33rd Critics' Circle Film Awards winners”“National Board of Review Current Awards”Bản gốc"Ron Howard Whaling Tale 'In The Heart Of The Sea' Casts Tom Holland"“'Spider-Man' Finds Tom Holland to Star as New Web-Slinger”lưu trữ“Captain America: Civil War (2016)”“Film Review: ‘Captain America: Civil War’”lưu trữ“‘Captain America: Civil War’ review: Choose your own avenger”lưu trữ“The Lost City of Z reviews”“Sony Pictures and Marvel Studios Find Their 'Spider-Man' Star and Director”“‘Mary Magdalene’, ‘Current War’ & ‘Wind River’ Get 2017 Release Dates From Weinstein”“Lionsgate Unleashing Daisy Ridley & Tom Holland Starrer ‘Chaos Walking’ In Cannes”“PTA's 'Master' Leads Chicago Film Critics Nominations, UPDATED: Houston and Indiana Critics Nominations”“Nominaciones Goya 2013 Telecinco Cinema – ENG”“Jameson Empire Film Awards: Martin Freeman wins best actor for performance in The Hobbit”“34th Annual Young Artist Awards”Bản gốc“Teen Choice Awards 2016—Captain America: Civil War Leads Second Wave of Nominations”“BAFTA Film Award Nominations: ‘La La Land’ Leads Race”“Saturn Awards Nominations 2017: 'Rogue One,' 'Walking Dead' Lead”Tom HollandTom HollandTom HollandTom Hollandmedia.gettyimages.comWorldCat Identities300279794no20130442900000 0004 0355 42791085670554170004732cb16706349t(data)XX5557367